[ /tv/ /rf/ /vg/ /a/ /b/ /u/ /bo/ /fur/ /to/ /dt/ /cp/ /oe/ /bg/ /ve/ /r/ /mad/ /d/ /mu/ /cr/ /di/ /sw/ /hr/ /wh/ /lor/ /s/ /hau/ /slow/ /gf/ /vn/ /w/ /ma/ /azu/ /wn/ ] [ Main | Settings | Bookmarks | Music Player ]

No.66948 Reply
File: Математика.jpg
Jpg, 21.06 KB, 350×263 - Click the image to expand
edit Find source with google Find source with iqdb
Математика.jpg
Предлагаю задавать здесь любые вопросы, касающиеся математики.
Полезные ссылки:
LaTeX pastebin: http://mathbin.net/
Если требуется что-то посчитать: http://www.wolframalpha.com/
Предыдущий: >>55775
>> No.66961 Reply
Головоломка.
Каждая точка некоторого топологического пространства обладает некоторой окрестностью, гомеоморфной евклидову пространству R^n. Что можно сказать про отделимость в этом топологическом пространстве?
>> No.66978 Reply
File: rena.jpg
Jpg, 120.99 KB, 704×400 - Click the image to expand
edit Find source with google Find source with iqdb
rena.jpg
>>66961
Ой какая прелесть. Вполне регулярность очевидно есть, а с нормальностью чего-то пока не придумывается.
А ещё есть первая аксиома счётности, но сепарабельности может не быть.
>> No.67007 Reply
File: Снимок.PNG
Png, 1.80 KB, 175×41 - Click the image to expand
edit Find source with google Find source with iqdb
Снимок.PNG
Кто разъяснит идиоту как взять интегралы с рациональными дробями. Книга передом мной, но я что-то не могу въехать по поводу Pn и Qm. Вот допустим есть интеграл на картинке. В книге написано, что такие дроби надо раскладывать. Как они вообще сравнивают эти дроби? Я не могу понять весь этот метод неопределенных коэффициентов. Хотя он самый самый простой.
>> No.67009 Reply
File: Scan10004.jpg
Jpg, 1915.39 KB, 3322×2497 - Click the image to expand
edit Find source with google Find source with iqdb
Scan10004.jpg
>>67007
Сначала получаешь правильную дробь, потом разлагаешь её на элементарные. Понятно?
>> No.67026 Reply
File: Photo-29.06.12,-17-02-08.jpg
Jpg, 265.61 KB, 960×720 - Click the image to expand
edit Find source with google Find source with iqdb
Photo-29.06.12,-17-02-08.jpg
Привет, матемач!
Нужна помощь. Вот смотри, стоит задача найти расстояние между джвумя кривыми на плоскости (пикрелейтед).
Делаем так - естественно определяем расстояние как наименьшее между точками кривых, строим функцию двух переменных и ищем ее наименьшее значение. Вторую кривую мы можем задать функцией, а вот первую - нет. Разрешив ее уравнение относительно одной переменной получаем две функции, отличающиеся только знаком.
Однако, если нарисовать пикчу, видим, что графики этих двух функций симметричны относительно начала координат, а значит, учитывая, что мы ищем не экстремум а лишь наименьшее значение, можно смело выбросить +- и вместо двух функций исследовать одну.
Проблема в том, что картинка и слово "очевидно" не будут строгим обоснованием моих действий. Как пояснить по хардкору так, чтобы до меня точно никто не подумал доебаться?
>> No.67028 Reply
File: Photo-29.06.12,-17-02-28.jpg
Jpg, 256.79 KB, 720×960 - Click the image to expand
edit Find source with google Find source with iqdb
Photo-29.06.12,-17-02-28.jpg
>>67026
Блин, пикчу перекосоебило. Починил.
>> No.67029 Reply
>>67009
За решение конечно спасибо, но я больше хотел и теорию почитать. Не понял 3 пункт (сумму). А вернее правый интеграл.
>> No.67034 Reply
>>67026
Я конечно не спец, но мейби тебе стоит повернуть кривую второго порядка используя угол уравнения прямой. Т.е. сделать так чтобы эта прямая стала твоей новой осью x, тогда тебе нужно будет просто найти минимум этой функции, а затем вернуться к старой системе координат.
>> No.67035 Reply
>>67034
Не, кривая все равно на две функции развалится, через функцию двух переменных там совсем просто все.
>> No.67042 Reply
>>67029
Дробь \frac{...}{qrp}, где q, r и p — линейные множители, ничто не мешает представить в виде \frac{α}{q} + \frac{β}{r} + \frac{γ}{p}.
Из того, что второе выражение должно быть тождественно равно первому, получаем:
\frac{αrp + βqp + γqr}{qr*p}
В комплексных числах любую дробь можно так разложить.
>> No.67043 Reply
>>67042
> \frac{αrp + βqp + γqr}{qr*p} = \frac{...}{qrp}
> αrp + βqp + γqr = ...
ffix.
>> No.67045 Reply
>>67026
Так, чего-то я подумал, а потом передумал, а потом опять подумал. Анон, таки делай методом Лагранжа. Дёшево и сердито, и думать не надо. Минимизируем расстояние до прямой при условии гиперболы. Там очень легко всё получается.

>>66961
Аноняша, сдаюсь, подсказывай. Кстати, а верно ли, что всякое локально компактное пространство паракомпактно?
>> No.67047 Reply
>>67045
Я уже решил, там симметричность легко аналитически доказывается d1(x,y) = d2(-x,-y), все хорошо обосновывается.
А кстати, как будет выглядеть с функцией Лагранжа? Как-то плохо себе представляю. Ну то есть чисто механически как написать знаю, а вот суть действий пока не понел.
>> No.67051 Reply
>>67029
Анон, мы просто берём дроби, при суммировании которых получается дробь с точно таким же как у нас знаменателем и полиномом нужной степени в числителе. А потом смотрим, какие A, B и С должны были быть сверху, чтобы всё совпало. Теперь всё понятно?
Причём для случаев как у тебя существование и единственность этих A, B и С следует из однозначности решения системы. А что там для случаев с комплексными и кратными корнями не знаю и разбираться лень.
>> No.67057 Reply
>>67035
Дак она и после поворота останется функцией двух переменных, однако там можно легко получить простую систему уравнений, т.к. искомый минимум будет в точке где градиент будет равен нулю
>> No.67062 Reply
>>67057
Няш, проясни, пожалуйста, письменно, а то я не совсем понимаю, о чем речь.
>> No.67070 Reply
Тут меня как-то логически переклинило. Помогите выйти из клина.
sqrt((-1)/(-2)) = sqrt(1/2) - определен на множестве действ. чисел. Минус сверху и снизу сокращается.

Но! sqrt((-1)/(-2)) = sqrt(-1)/sqrt(-2). Ни верхняя, ни нижняя части на действительных числах неопределены.

ЧЗХ?

Конкретно аналогичная ситуация возникла при попытке не потерять ОДЗ при решении уравнения.
>> No.67071 Reply
File: Scan10004.jpg
Jpg, 1147.05 KB, 3322×2497 - Click the image to expand
edit Find source with google Find source with iqdb
Scan10004.jpg
>>67051
Я ничего не понял, пик.
> \frac{...}{qrp}, где q, r и p — линейные множители, ничто не мешает представить в виде \frac{α}{q} + \frac{β}{r} + \frac{γ}{p}
Это вообще ад, что за frac и т.д. Можно на просто языке?
>> No.67074 Reply
>>67070
Корень из 1/2 определен на множестве всех действительных чисел? Ну дела.
Няш, сформулируй так, чтобы было понятно, что именно тебя смущает.
>> No.67077 Reply
>>67071
Дробь же! Вот тебе вольфрамальфа, например:
http://www.wolframalpha.com/input/?i=\frac{1%2Bx}{1-x}

Поверь мне, latex для математики много лучше, чем какой-нибудь MathML.
>> No.67078 Reply
>>67071
По твоему пикрелейту, от 2x^4 + 2x^3 - 41x^2 + 20 просто оторвали -x^2 + 20, 2x^4 + 2x^3 - 40x^2 же осталось.
>> No.67089 Reply
>>67070
Ну дык у формулы \sqrt{xy}=\sqrt{x}\sqrt{y} есть некая область применения. В действительных числах никто не обещал, что она верна для всех x и y. Только для неотрицательных.

Вообще, "формул" не бывает. Ну то есть не в том смысле, в котором ты пытаешься понимать. А бывают теоремы. "Теорема: если x и y - положительные числа, то \sqrt{xy}=\sqrt{x}\sqrt{y}." Эта теорема считается доказанной в школьном курсе, её можно применять. Если ты в каждой строчке можешь указать, какой теоремой ты пользуешься, то задача решена верно. Обобщения на случай, когда x или y отрицателен, не доказываются, и поэтому применять их нельзя. Поэтому равенство
> sqrt((-1)/(-2)) = sqrt(1/2)
правильно доказать внутри данной задачки не удаётся. Вроде как всё.
>> No.67102 Reply
>>67070
> Но! sqrt((-1)/(-2)) = sqrt(-1)/sqrt(-2)
Просто это выражение не тождество: оно верно только для положительных чисел.
К сожалению, комплексные числа тут мало помогут: с ними корень — не функция.
>> No.67104 Reply
>>67102
> не функция
Имеется в виду не числовая функция.
>> No.67146 Reply
>>67078
Так я спрашиваю - какого фига оторвали.
>> No.67161 Reply
>>67146
Нужно было получить правильную дробь, в которой степень числителя меньше степени знаменателя. Нужно избавиться от 2x^4 и 2x^3. От исходного 2x^4 + 2x^3 - 41x^2 + 20 отрываем 2x^4 и всё, что нужно, чтобы оторванный кусок делился на знаменатель без остатка. То есть, наш кусок: 2x^4 + 2x^3 - 40x^2. Получилось, что и от 2x^3 мы тоже избавились. Осталась правильная дробь (-x^2 + 20)/(x^3+x^2-20x), которую можно разложить на элементарные.
>> No.67232 Reply
Доброняша, выручай. Понять не могу, как решать.
Ищем расстояние между плоскостью
3x + 4y + 12z = 288 и эллипсоидом
x^2/96 + y^2 + z^2 =1
Решение в лоб, как я эт о делал раньше не оче проходит, решать систему из 4 нелинейных уравнений это чересчур. Wat do?
>> No.67234 Reply
>>67232
> Wat do?
Найти параллельную касательную плоскость к эллипсу.
>> No.67235 Reply
>>67234
оиду.
>> No.67238 Reply
File: f1127.JPG
Jpg, 3.59 KB, 182×47 - Click the image to expand
edit Find source with google Find source with iqdb
f1127.JPG
>>67235
Расстояние от точки до плоскости пикрелейтед.
Ищешь минимум методом Лагранжа.
>> No.67239 Reply
>>67238
Мне-то зачем это говорить? Я просто эллипсы и эллипсоиды перепутал. Метод Лагранжа здесь не нужен.
>> No.67240 Reply
Как разложить x^3 на отрезке [0;½] по косинусам нечётных дуг?
>> No.67243 Reply
>>67239
Извини, не тебе хотел ответить. Но да, твоё решение попроще.
>> No.67245 Reply
>>67243
А если подставить произвольную точку эллипсоида в формулу расстояния и посмотреть, как раскрывать модуль, то будет точно таким же. Функция Лагранжа будет выглядеть почти также, как уравнение касательной.
>> No.67251 Reply
>>67234
>>67238
Спасибо, аноны!
С вашего позволения, еще один вопрос, на этот раз про функцию Лагранжа.
Допустим, стоит задача поиска условного экстремума
f:R^n -> R --> min/max
φ1(x1...xn) = 0
..
φm(x1...xn) = 0
Построили функцию Лагранжа, нашли мы ее критические точки, записали форму второго дифференциала. Она, допустим, по всему пространству получилась знаконеопределенной. И вот тут начинается самое интересное - говорим хуй с ним со всем пространством, выясним знакоопределенность второго дифференциала на множестве точек, в которых дифференциал функций уравнений связи равен нулю
M = {(x1,..,xn) | dφi = 0 i=1..m}
Что такого охуительного в этих точках, что нам достаточно знакоопределенности формы на M для констатации условного экстремума?
>> No.67258 Reply
>>67251
Ну это, анон, я так понимаю, как и у функции одной переменной. Просто, чтобы производная была равна нулю для экстремума недостаточно. Нужно чтобы вторая производная не меняла знак в некоторой окрестности. Как у x^3 в нуле. Такого объяснения тебе достаточно?
>> No.67261 Reply
>>67258
Не совсем, это-то я как раз понимаю. Я не понимаю, почему нам второй дифференциал интересен не на всем пространстве, а только на {(x1,..,xn) | dφi = 0 i=1..m}, и почему именно такие точки, что dφi = 0.
Нам просто метод Лагранжа без теории выдавали, хотет на пальцах разобраться, в чем тут дело.
чур линии спрашивать - ну одобряет же.
>> No.67406 Reply
File: Scan10005.JPG
Jpg, 3239.16 KB, 3336×2864
edit Find source with google Find source with iqdb
Scan10005.JPG
File: ИП.jpg
Jpg, 58.19 KB, 483×409
edit Find source with google Find source with iqdb
ИП.jpg

>>67261
Размышляя над тем, что тебе ответить, вдруг понял, что сам ничего не понимаю.
На случай если тебя вдруг всё ещё интересует этот вопрос:

На пальцах.
Сначала объясню, как можно найти экстремумы без λ, так будет гораздо проще.
Допустим, мы ищем минимум функции 15.40 m+n переменных, при условии 15.41. Если у нас все функции хорошие, то найдутся такие φi (не те φi, которые у тебя. То что у тебя было φi, у нас Fi), что 15.43 будет решеним 15.41. Подставляем эти φ в f и получается, что нам нужно найти не условный экстремум, а обычный, и переменных только n, а не m+n).
Но выразить yi в явном виде через xi может быть невозможно. Поэтому идём дальше.
Чтобы в точке был экстремум нужно, чтобы 15.46. Заметь, что dyi - дифференциалы функций, а не независимых переменных и yi в явном виде мы найти не можем, поэтому, как дифференциал выглядит, тоже не знаем, и в чистом виде использовать это равенство не можем.
Чтобы найти dyi, мы продифференциируем систему 15.41 и получим 15.47. Из 15.47 мы можем выразить dyi через dxi и подставить всё в 15.46.
15.46 у нас превращается в 15.48, а так как переменные xi независимы, 15.48 равносильно равенству нулю коэффициентов Ai.
То есть чтобы найти экстремум нам остаётся только решить систему 15.49.

К твоему случаю это тоже применимо. На рисунке я начал решать его этим способом. z принимаем за функцию от x и y.

Теперь метод Лагранжа.
Мы делаем всё точно тоже самое. Просто вместо того, чтобы выражать одни дифференциалы через другие, мы просто домножаем 15.47 на λi и складываем все уравнения с 15.46, а потом приравниваем коэффициенты при дифференциалах к нулю, чтобы равенство выполнялась. Если функции хорошие, то всё проходит.
Ты наверное спросишь, зачем домножать всё на λ, почему нельзя просто так сложить. Чтобы могли выполниться условия 15.41, их мы тоже добавляем в систему.
Вот так. Картинки из Ильина-Позняка.
>> No.67409 Reply
>>67261
Ещё можно объяснить через градиенты. Можешь на википедии посмотреть.
> не на всем пространстве
Мы ведь ищем локальный экстремум. Кроме окрестностей точек нас вообще ничего не интересует.
>> No.67412 Reply
File: 1276170260054.jpg
Jpg, 43.33 KB, 485×654 - Click the image to expand
edit Find source with google Find source with iqdb
1276170260054.jpg
>>67009
Блядь, что это такое? Как ты из первого знаменателя получил второй? Чем дальше я продвигаюсь во временном пространстве без матана, тем сложнее он для меня становится (тупею). Нахуя там dx писать всё время?
Как жаль, что я безвольное хуйло и уже никогда не освою так ненавистный мною матан.
>> No.67447 Reply
>>67412
Это же всё первый курс.
Советую покопаться в основах, тогда и станет очевидным и смысл присутствующего dx, и множители Лагранжевы, и те же контурные интегралы.
>> No.67465 Reply
Аноны, посоветуйте книжек за первый курс. Я его как-то сдал, ничего не зная, хочу за лето нагнать.
>> No.67480 Reply
>>67406
Ох как. Спасибо большое за ответ, доброанон. Пока не могу сказать, что все понел, но это уже охуенно.
> Если у нас все функции хорошие, то найдутся такие φi (не те φi, которые у тебя. То что у тебя было φi, у нас Fi), что 15.43 будет решеним 15.41
Хорошие функции - это я правильно понимаю - тут речь о выполннении условий страшной-престрашной теоремы о неявной функции, да?
>>67409
> Мы ведь ищем локальный экстремум. Кроме окрестностей точек нас вообще ничего не интересует.
Точно так, но нас даже не вся окрестность почему-то интересует, а только вон то самое ее подмножество.
>> No.67483 Reply
File: 20120621_120908.jpg
Jpg, 966.68 KB, 2048×1536 - Click the image to expand
edit Find source with google Find source with iqdb
20120621_120908.jpg
>>67465
двачую
>> No.67485 Reply
>>67483
билетрандом, как пикрандом, так что на всю его простоту не пиняйте
>> No.67495 Reply
>>67480
> Хорошие функции - это я правильно понимаю - тут речь о выполннении условий страшной-престрашной теоремы о неявной функции, да?
Ну да. Они должны быть непрерывно дифференцируемы и одно условие не должно вытекать из другого. А если условие всего одно, оно просто не должно быть тождеством.
> Точно так, но нас даже не вся окрестность почему-то интересует, а только вон то самое ее подмножество.
Само собой, мы же ищем условный экстремум, то есть экстремум на этом самом подмножестве.
>> No.67496 Reply
>>67465
>>67483
Ильин-Позняк, Фихтенгольц, Зорич - говорят хорошие. Если не понимаешь чего-то в одном учебнике, загляни в другой, может быть там понятнее. Могу сказать, что Ильин-Позняк гораздо длиннее и гораздо проще того, по чему учились мы. А если пытаешься разобраться один, и рядом нет ботана или препода, то главное понятность.
>> No.67526 Reply
А сложно ли ввести поддержку latex'а на доброчане?
Может попросим админов?
>> No.67546 Reply
>>67526
Все равно диаграмм не будет. Без них латех не нужен.
>> No.67547 Reply
File: Безымянный.jpg
Jpg, 3.22 KB, 169×48 - Click the image to expand
edit Find source with google Find source with iqdb
Безымянный.jpg
Анон, помоги решить интегральное уравнение. Прошу тебя, мне очень надо.
>> No.67551 Reply
>>67547
x(t) - константа, что ли?
>> No.67552 Reply
>>67551
Произвольная функция, насколько я понимаю.
>> No.67555 Reply
>>67552
Твой интеграл - это константа. Получается, что x(t) - это единица минус лямбда умножить на константу, то есть тоже константа.
>> No.67558 Reply
>>67555
Не понимаю. Если приводить к классическому виду ур-ия Фредгольма, то будет фи от х = 1 - интеграл с ядром (t+1). Константа получается потому что в ядре нету х?
>> No.67564 Reply
>>67558
> Константа получается потому что в ядре нету х?
Константа получается потому что в ядре нету t.
>> No.67567 Reply
Какие учебники можете посоветовать по топологии?
Как вам учебник Виро?
>> No.67619 Reply
File: Безымянный.png
Png, 6.49 KB, 353×225 - Click the image to expand
edit Find source with google Find source with iqdb
Безымянный.png
У меня паника, строю график по пику - говорят не правильно, строил и вручную несколько раз, и на пк строил.
Анончик, помоги нормально построить, включая точку пересечения оси y. У меня получается дуга, которая из 2 в 1 идет четверти.
>> No.67630 Reply
>> No.67639 Reply
>>67630
Опа, а почему у меня получалась просто в одной четверти? Поможешь составить точечно ее?
>> No.67649 Reply
>>67639
Точечно - это по точкам?
Просто вырази из 2-го равенства t через y, подставь в x, получатся 2 функции, нарисуй их.
>> No.67651 Reply
>>67639
> а почему у меня получалась просто в одной четверти?
Скорее всего где-то сделал неравносильное преобразование, когда преобразовывал синусы в арксинусы или что-то типа того.
>> No.67764 Reply
>>67649
Просто я подставлял значения, допустим pi/2 вместо t и смотрел каждую точку, но это неверно.
>> No.67778 Reply
>>67764
про pi/2 <= t <= pi я как раз и забыл. Возможно ты прав и так и есть?
>> No.67821 Reply
Матемач, хорошего дня! Я снова на связи.
Вот допустим у меня задача - найти экстремумы функции y неявно заданной уравнением F(x1..xn,y) = 0.
Я могу сказать, что задача эквивалентна исследованию на условный экстремум функции u = y с ограничением F(x1..xn,y) = 0?
>> No.67823 Reply
>>67821
> найти экстремумы функции y, неявно заданной уравнением F(x1..xn,y) = 0.
В самом важном месте запятую забыл, ну дела.
>> No.67830 Reply
>>67778
но подставь 0,0 Мне надо узнать, где на этом санном графике будет точка пересечения оси yю
>> No.67851 Reply
>>67821
Ну да, почему бы и нет. Что u зависит только от y значения не имеет.
>>67830
Чтобы найти точки пересечения с OY, просто подставь x=0 в свою систему. Получится, что t=sint, но t⩾π/2, значит t>1, значит t>sint, пересечения с OY нет. И вообще у t=sint всего один корень t=0.
>> No.67880 Reply
>>67851
Бред, пересечение с осью должно быть.
>> No.67882 Reply
>>67880
x=0 - это и есть пересечение с осью.
>> No.67907 Reply
>>67882
Это я понимаю, но как правильно брать по y, у меня 2 разных результата получает, 1 в нуле, другой, я думаю, понятно где.
>> No.67915 Reply
Котоны, поясните за матпакеты.
Wolfram Mathematica vs Maple vs MatLab
Кто чем пользуется? Что лучше? Какие плюсы и минусы?
>> No.67916 Reply
http://dobrochan.ru/rf/res/300564.xhtml#i300932
Гуманитароблядки как всегда смачно отсосали, зафейлив раскрыть скобки в логическом выражении, соответственно провалив попытку понять смысл статьи закона, который говорит, не что лоль можно ебать, а что даже некоторых нелоль ебать нельзя.
>> No.67917 Reply
>>67916
Это их проблемы, им же математика не пригодится. А вообще, проводить разделение на стаю гуманитариев и стаю математиков, при этом всячески гнобить чужую стаю, - признак обезьяньего мышления.
>> No.67918 Reply
Интерполяция многочленами, многочлен Лагранжа, полиномиальная интерполяция — одно и то же? И ещё, почему квадратичная интерполяция менее точна, нежели Лагранжева?Подозреваю, из-за того, что там только параболически, а здесь 3+ степень, правильно мыслю?
>> No.67919 Reply
>>67916
Самое замечательное, что мелкой говотне 13 лет и это другой пункт, в котором о половозрелости ничего не сказано.
>> No.67935 Reply
>>67916
Ты на доброчане, а не на мочепараше, изволь нормально разговаривать.
>> No.67936 Reply
>>67915
> Wolfram Mathematica vs Maple vs MatLab
Проприетарщина нинужна. Юзай свободный софт: Scilab, GNU Octave, Sage, FreeMat, Maxima. Выбор на любой вкус. Я бы предложил для численных расчетов Scilab или GNU Octave, для символьных - Maxima. Ну а если хочешь не ограничиваться возможностями этих систем, есть более универсальный вариант - python + различные библиотеки (numpy, scipy, matplotlib, sympy, тысячи их). В Sage это все это (и не только это) объединено в единый интерфейс с синтаксисом от python. Сам юзаю последний вариант.
>> No.67937 Reply
>>67936
> Проприетарщина нинужна.
Опенсорсные кривые недоподелки нинужны.
>> No.67939 Reply
>>67937
Не забудь заплатить за софт, няшик.
>> No.67942 Reply
>>67939
Я скачаю с торрентов и меня ничто не остановит.
>> No.67944 Reply
>>67939
Не забудь взять у мамы денег на проезд, няшик.
Триста долларов за полный комплект вычислительного ПО - дёшево.
>> No.67947 Reply
>>67939
А я студент, у меня скидки и бесплатноништяки.
>> No.67959 Reply
Проприетрщина плоха даже не тем, что платная, а тем, что несвободная. Какие ОС поддерживать решает господин, когда выпускать новую версию и что в нее включать решает господин, посмотреть код алгоритмов - еще чего захотел, добавить свой функционал - в лучшем случае пили костыли в виде плагинов всяких. Добро пожаловать на иглу. Нет, зачем вы нужны с такими условиями. Я уж не говорю про то, что в образовании и публичной науке использовать приприетарщину по большому счету не следует вообще. Ну ладно, каждый для себя решает сам, главное чтобы это не было навязыванием.
> у меня скидки и бесплатноништяки
Бесплатный сыр от проприетарщиков.
>> No.67960 Reply
>>67959
Но ты навязываешь! Весь пост пропитан рекламой опенсорса!
>> No.67962 Reply
>>67959
> Какие ОС поддерживать решает господин, когда выпускать новую версию и что в нее включать решает господин
Деление на "рабов" и "господ" устарело. Здесь просто торговля. Когда мне нужен телевизор, я не пытаюсь сделать его в домашних условиях. Я просто иду и покупаю его. Мне неинтересно как он устроен, мне хочется, чтобы он показывал мне мультики.
> добавить свой функционал - в лучшем случае пили костыли в виде плагинов всяких
Когда ты, лично ты, добавлял новый функционал в сайлаб?
> Бесплатный сыр от проприетарщиков.
Истинно так. Для этого, знаешь ли, и придуманы скидки - чтобы привлекать клиентов.
>> No.67982 Reply
>>67960
> Но ты навязываешь! Весь пост пропитан рекламой опенсорса!
Я информирую, а не навязываю.
>>67962
> Деление на "рабов" и "господ" устарело. Здесь просто торговля. Когда мне нужен телевизор, я не пытаюсь сделать его в домашних условиях. Я просто иду и покупаю его. Мне неинтересно как он устроен, мне хочется, чтобы он показывал мне мультики.
Согласен. И когда твой телевизор сломается и ты захочешь купить новый - ты просто пойдешь и купишь любой другой телевизор. Однако со сложным софтом это не совсем так. Однажды купив, ту уже перестаешь быть свободным в дальнейшем выборе. К примеру однажды некто подсел на винду/мс-офиc/матлаб/фотошоп/етц. Потом захотел слезть, а ни тут-то было - слишком много уже вложено, слишком много завязано. При использовании сложного софта возникает зависимость, для однажды подсевшего теряется симметрия и единственный поставщик становится монополистом, диктующим свои условия. И это ненормально. Это опасное явление. Это можно сравнить с наркотиком (только не надо придираться к аналогии, это просто анология). И поэтому наркотики везде запрещены. Я не призываю запретить проприетарный софт, я лишь за то, чтобы каждый понимал последствия, был информирован об особенностях проприетарного софта. И чтобы если человек выбирает проприетарный софт, он делал это осознанно и расчетливо, а не просто подумав "в падлу искать и изучать свободные программы да и вообще говорят что они все кривые, лучше матлаб с торрентов качну". На проприетарном софте как на сигаретах надо писать "осторожно, использование этим софтом может имеет следующие особенности: ...". Да только кто читать будет, бесправные для пользователя EULA же некто читает.
> Когда ты, лично ты, добавлял новый функционал в сайлаб?
Никогда. Но возможности есть.
>> No.67983 Reply
>>67982
> Однажды купив, ту уже перестаешь быть свободным в дальнейшем выборе. К примеру однажды некто подсел на винду/мс-офиc/матлаб/фотошоп/етц. Потом захотел слезть, а ни тут-то было - слишком много уже вложено, слишком много завязано.
Ты так говоришь, будто опенсорсность того или иного поделия помогает с него слезть в случае надобности.
> "в падлу искать и изучать свободные программы да и вообще говорят что они все кривые
Но ведь почти все опенсорсные программы кривые. Даже в gcc не могут устранить совершенно детских багов http://habrahabr.ru/post/147128/ . Кроме того, большинство опенсорсных программ - это "кастрированные" кальки нормальных, хороших проприетарных софтин.

Хочешь подискутировать о своей вере в опенсорс - создай тред в /lor, поговорим.
>> No.67986 Reply
>>67935
анальный баттхерт добропетушка
>> No.67988 Reply
File: Кот_Данко.jpg
Jpg, 426.86 KB, 529×670 - Click the image to expand
edit Find source with google Find source with iqdb
Кот_Данко.jpg
Няши, не угоняйте мой тред, пожалуйста. Он про математику.
>> No.67989 Reply
>>67944
> Триста долларов за полный комплект вычислительного ПО - дёшево.
Триста долларов за матпакет, который устареет, когда выйдет следующая версия, и ее придется покупать снова - слишком дохуя

да даже 140$ для студентов - дохуя
>> No.68041 Reply
File: 1322604360743.jpg
Jpg, 113.93 KB, 800×600 - Click the image to expand
edit Find source with google Find source with iqdb
1322604360743.jpg
>>67918
Не игнорируйте, няши!
>> No.68044 Reply
>>66978>>67045
Конечно, я немного запоздал. Болел.
T0, T1 удовлетворяет.
T2, T3, T3.5, T4 не удовлетворяет (контрпример - склейка двух прямых по интервалам).
>> No.68047 Reply
>>67918
Здесь и далее говорим о функциях одной переменной.
> Интерполяция многочленами
Означает приближение данной функции f(x) (возможно, не известной для всех x) некоторым многочленом g(x).
> многочлен Лагранжа
Интерполяционный многочлен Лагранжа.
Один из конструктивных вариантов интерполяции. Из распространенных есть совершенно эквивалентный многочлен Ньютона, строящийся по другому алгоритму. И еще был многочлен, кажется, Эрмита, который строился не только по значениям в точке, но и по производным произвольного порядка.
> полиномиальная интерполяция
То же, что и интерполяция многочленами.
> И ещё, почему квадратичная интерполяция менее точна, нежели Лагранжева?
Прозреваю, что под квадратичной интерполяцией подразумевается интерполяция сплайнами - "распиливание" отрезка, на котором интерполируется f(x) на n подотрезков и представление f(x) на каждом из них в виде своего полинома g_n(x). Этот вариант не менее точный, он, кхм, другой. Иногда дающий более хорошее приближение.
>> No.68052 Reply
File: Квадратичная-инте...
Pdf, 0.22 KB, 595×842
Квадратичная-интерполяция.pdf
File: 12545584487849.jpg
Jpg, 218.42 KB, 1366×768
edit Find source with google Find source with iqdb
12545584487849.jpg

>>68047
> Прозреваю, что под квадратичной интерполяцией подразумевается интерполяция сплайнами
Таки нет, вот методичка.
>> No.68054 Reply
>>68052
Бог с ним, тогда это действительно просто интерполяция квадратным трехчленом. В методичке указано даже, зачем оно - для численного обнаружения минимума функции. Здесь вторая степень выглядит вполне оптимально.
>> No.68107 Reply
Анон,

Теория групп. Чего читать [i]пятиклашке[/i]?
>> No.68128 Reply
File: full-c8f28f5b63a054e65dbed977c361739e.png
Png, 84.26 KB, 360×360 - Click the image to expand
edit Find source with google Find source with iqdb
full-c8f28f5b63a054e65dbed977c361739e.png
>>68044
Ты меня пугаешь. У меня так не получается склеить, чтобы ни в одной точке не нарушилась локальная евклидовость.
Алсо, неужели нижеследующее неверно?
Вот у нас две точки x,y в нашем пространстве, U - такая открытая окрестность x, что есть гомеоморфизм f:U->R^n. Найдем открытый шар G вокруг f(x) такой, что замкнутый шар H того же радиуса вокруг f(x) не содержит f(y). Тогда f^{-1}(G) и f^{-1}(R^n\H) - непересекающиеся окрестности x и y.
>> No.68136 Reply
Здравствуй, /u.
Хочу за лето подтянуть (читай: по-новой изучить) МСС(ж) и уравнения в частных производных. Поэтому реквестирую учебников по этим дисциплинам.
>> No.68139 Reply
File: рисунок.png
Png, 6.07 KB, 355×355 - Click the image to expand
edit Find source with google Find source with iqdb
рисунок.png
>>68128
> У меня так не получается склеить, чтобы ни в одной точке не нарушилась локальная евклидовость.
Картинка. Красненькая окрестность x1 гомеоморфна прямой (как и синенькая у y1). Но каждая окрестность x1 пересекает какую-то окрестность y1.
> Алсо, неужели нижеследующее неверно?
А если y не принадлежит U? Может быть так, что каждая окрестность, содержащая x и y не гомеоморфна R^n. Отсюда следует, только что у x есть окрестность, не содержащая y, и у y есть окрестность, не содержащая x, (удовлетворяет аксиоме Фреше).
>> No.68163 Reply
File: Sovposter11.jpg
Jpg, 72.36 KB, 658×480 - Click the image to expand
edit Find source with google Find source with iqdb
Sovposter11.jpg
>>68139
А да да даааааа, R^n\H может не быть пересечением с U никакого множества, содержащего y.
Ну хорошо, но это не отменяет вопроса - если заранее потребовать хаусдорфовость, то будет ли сразу нормальность?
>> No.68190 Reply
Анон, я тут решил просветиться Ван-дер-Варденом и столкнулся с проблемами буквально в самом начале. Мне интересно, будет ли действовать индуктивный метод доказательства некоего свойства для множества натуральных чисел, заданного от 3 до бесконечности? Просто в аксиомах Пеано из которых этот метод выводится, говорится о множестве натуральных чисел от 1 до бесконечности...
И еще, когда вводятся 0 и отрицательные числа, используется какой-то хитрый метод записи типа (a+b,b) или (b,a+b). Я понял, что b - произвольное натуральное число и другие уж совсем очевидные вещи. Но зачем мы представляем числа парами и что обозначает запятая внутри скобки?
>> No.68202 Reply
>>68190
1)Будет действовать для любого индуктивного множества. Не помню как это описано в ван дер Вардене. Верно, например, следующее есть множество X, F его биективно отображает в N или, если X конечно, в начальный отрезок N (1..n).
И если выполнено P(F(-1)(1)) F(-1) - обратная функция
и
P(F(-1)(n)) => P(F(-1)(n+1))
то P верно на всем Х т.к. можно обозначить P'(n) = P(F(-1)(n))
ну а тогда P' удовлетворяет необходимым условиям.
вроде нигде не ошибся надеюсь
>> No.68203 Reply
>>68190
> Я понял, что b - произвольное натуральное число
да
> Но зачем мы представляем числа парами и что обозначает запятая внутри скобки?
Потому что теория множеств гарантирует существование натуральных чисел, и существование декартова произведения любого множества, поэтому мы водим отношение эквивалентности на декартовом произведении натуральных чисел, таким вот способом. Одним словом целое число это класс эквивалентности.
>> No.68283 Reply
>>68203
Так, я приблизительно понял, что мы вводим пары натуральных чисел и вводим некие правила работы с ними для возможности декартовых произведений множеств. Еще Ван-дер-Варден говорит, что выполняются некие правила (3), (4), (5), (8), (9), (10), (12), (13), (14), (15) для определений работы с парами чисел. Что это за правила? Или я не так истолковал написанное?
>> No.68308 Reply
>>68283
> мы вводим пары натуральных чисел и вводим некие правила работы с ними для возможности декартовых произведений множеств.
Наоборот у нас есть декартово произведение натуральных чисел, мы на нем вводим отношение эквивалентности(b,b + a) ~ (c, c + a) и (a + b, b) ~ (a + c, c)изначально у нас нет даже разности и фактор-множество NxN по этом отношению будет целыми числами. В данном случае сложениепокомпонентное будет фактор-операцией, т.е. для сложения двух классов достаточно сложить по элементу из них и посмотреть в каком классе элементы окажутся.
> Что это за правила?
За 2 страницы до этого они описаны (у меня на 21й странице)
>> No.68453 Reply
Добрый вечер, матемач!
Вот допустим у меня есть числовая случайная величина, заданная функцией плотности.
Как мне найти функцию плотности ее квадрата?
>> No.68467 Reply
>>68453
А подумать?
Если значение квадрата x, то это соответсвует двум начениям самой функции +-sqrt(x), соответственно плотность квадрата в этой точке p(sqrt(x))+p(-sqrt(x)).
>> No.68468 Reply
>>68467
Спасибо, анончик. Я правда думал, но почему-то тупил и никак не мог решить, что подставлять - квадрат или корень.
>> No.68496 Reply
Доброе утро, матемач! Помоги с задачей.
Есть числовая случайная величина, имеющая непрерывное распределение.
Взяли два ее семпла независимо друг от друга, один оказался меньше другого. Какова условная вероятность того, что третий семпл окажется меньше большего?
Какой-то Омск, а не задача, как к ней подступиться не понимаю. Во всяком случае, пока ничего умнее шутки про вероятность встретить Годзиллу не придумал. Я так понимаю, что это уже статистика а не тервер, так ведь?
>> No.68505 Reply
>>68496
Напомнило вот это: http://www.braingames.ru/?path=comments&puzzle=507
Оккупанты, известным только им образом, выбирают два различных вещественных числа и записывают их на двух бумажках. Затем предлагают Мегамозгу выбрать любую бумажку, посмотреть на написанное там число и угадать, больше число на другой бумажке или меньше. Докажите, что у Мегамозга есть стратегия, которая позволит ему угадать с вероятностью больше 50%.
>> No.68520 Reply
>>68308
Спасибо, теперь вроде понял.
>> No.68523 Reply
>>68505
Охренеть, вот это действительно омская задача.
А у нее точно есть решение? А то там люди на полном серьезе знают правильный ответ на вопрос "почему канализационные люки круглые?".
>> No.68533 Reply
>>68467
Нет, по-прежнему не понимаю ничего.
Вот допустим у нас величина принимает значения из [0,1].
Тогда интуитивно ожидается, что матожидание ее квадрата будет меньше, чем матожидание ее самой. Но при подстановке sqrt(x) в функцию плотности - получается больше. Та же история и со значениями из [1;+inf) - матожидание квадрата интуитивно кажется должно быть больше соответствующего значения самой величины, а получается наоборот.
Няша, помоги склеить этот пазл.
>> No.68534 Reply
File: nipah.png
Png, 77.40 KB, 336×280 - Click the image to expand
edit Find source with google Find source with iqdb
nipah.png
>>68496
Именно шутка про Годзиллу эта задача и есть.

У тебя три независимых одинаково распределенных случайных величины X, Y и Z. Из непрерывности следует, что пара равных среди них найдётся лишь с вероятностью нуль. Таким образом, предлагается вычислить вероятность события {(X<Y и Z<Y) или (Y<X и Z<X)}. Оно распадается на два равновероятных несовместных события: {Y - наибольшая из трёх} и {X - наибольшая из трёх}. Очевидно, событие {Z - наибольшая из трёх} имеет ту же вероятность, несовместно с этими двумя, и сумма всех трёх имеет вероятность 1. Значит, вероятность каждого равна 1/3, и ответ на задачу 2/3.

Логично?
>> No.68538 Reply
>>68534
Удивительно! Спасибо, мудрый анон!
Да, я несколько нечотко сформулировал условие -
> один оказался меньше другого
Тут мы знаем, какой именно (положим Y < X), поэтому и вопрос про условную вероятность. Проверь, пожалуйста, правильно ли я дальше сам делаю.
По определению условной вероятности:
P(Z<X | Y<X) = P((Z<X)^(Y<X))/P(Y<X)
Здесь P((Z<X)^(Y<X)) - очевидно вероятность события (Z меньше всех) P=1/3
В знаменателе P(Y<X), ну тут 1/2,
В итоге (1/3)/(1/2) = 2/3
мамочки, что ж я на экзамене буду делать, если мне такое попадется
Одие еще вопрос хотелось прояснить - равновероятность событий Z<X Z<Y, Y<Z Y<X, X<Z X<Y откуда вытекает. Вроде очевидно, но обосновать как-то не получается.
>> No.68545 Reply
>>68523
Есть решение. Я например ее решил, отправил на проверку, оказалось правильно. Довольно долго думал.
>> No.68555 Reply
>>68538
Если вдруг экзаменатор не захочет верить в очевидность равновероятности этих событий (на мой вкус это вполне здоровое проявление вероятностной интуиции), в крайнем случае можно выписать интегралы Стилтьеса, определяющие эти вероятности через совместное распределение этих величин, имеющей, в силу условия, вид dF(x)dF(y)dF(z), где F - непрерывная монотонно растущая от 0 до 1 функция распределения кого угодно из них, у всех одинаковая, и перестановкой пределов интегрирования допустимой в силу стандартной формулировки теоремы Фубини для интеграла Лебега на абстрактном пространстве доказать равенство этих интегралов.
>> No.68561 Reply
>>68555
Няша, расскажи, а зачем нам интеграл Лебега здесь?
Я не знаток, но один мудрый человек мне говорил, что в нашем частном случае непрерывной случайной величины мы можем использовать риманов, и я как-то не вижу тому препятствий.
Мне сейчас не совсем понятно что и по какому многообразию (это правильный термин?) мы интегрируем.
Я так понимаю, у нас трехмерное пространство, на котором задана некоторая непрерывная функция плотности. Нам нужно доказать, что-то вроде "масса множеств точек, задаваемых уравнениями x,y < z; y,z<x; x,z < y одинакова". Не могу сообразить пока, как это хоть сколько-нибудь строго сформулировать.
>> No.68617 Reply
> масса множеств точек, задаваемых уравнениями x,y < z; y,z<x; x,z < y одинакова
Да, так и есть. И это множество обрисовывается пределами интегрирования, и по нему интегрируется тождественная единица.
> в нашем частном случае непрерывной случайной величины мы можем использовать риманов
Здесь будет интеграл Римана-Стилтьеса (который, конечно, в данном случае существует), но он не сводится к обычному интегрированию даже в смысле Лебега по мере Лебега, потому что не всякая непрерывная величина абсолютно непрерывна. Теорему Фубини для интеграла Римана-Стилтьеса никто нигде не доказывает, а стандартную теорему Фубини для интеграла Лебега по произвольной мере (в том числе по мере Стилтьеса) мы можем считать известной и применить. Вот такое занудство я в этом вижу. Но если вам говорят о таких вещах не думать - не думайте.
>> No.68630 Reply
File: MSP13841a27c570dhcb458i0000473ehb164abgd2e2.gif
Gif, 0.94 KB, 125×46 - Click the image to expand
edit Find source with google Find source with iqdb
MSP13841a27c570dhcb458i0000473ehb164abgd2e2.gif
Как можно решить без перехода к кубическому уравнению пикрелейтед?
>> No.68646 Reply
File: easy.jpg
Jpg, 5.34 KB, 229×220 - Click the image to expand
edit Find source with google Find source with iqdb
easy.jpg
>>68630
Предварительно вынести корень из 3-x? Ну со всеми предосторожностями, разумеется. Как вариант - увидеть его в полученном кубическом уравнении и после этого элементарно найти два других.
>> No.68647 Reply
>>68646
Лол, прикольная очепятка. Вертелось в голове слово "корень", в третьем предложении заменил на "его", хотя это в совсем другом смысле "корень"!
>> No.68671 Reply
>>68617
Ну тут я опять нечотко высказался. С нашим уровнем подготовки мы безнадежно потонем в теории меры. Поэтому, конечно, никаких общих случаев не рассматривают. Только абсолютно непрерывные либо дискретные распределения, так что там можно считать функцию абсолютно непрерывной.
В любом случае, спасибо за помощь, хоть что-то я понел.
>> No.68689 Reply
>>68671
Тогда не задумывайся об этой фигне и спокойно переставляй пределы интегрирования.
>> No.68691 Reply
File: eb8oxsdx6539.jpg
Jpg, 5.18 KB, 200×200 - Click the image to expand
edit Find source with google Find source with iqdb
eb8oxsdx6539.jpg
Аноны, я не понимат! Почему вероятность того, что три раза подряд выпадет орел 1 к 6, но при этом вероятность выпада орла на монетки все равно 1 к 2???
>> No.68693 Reply
>>68691
1 к 8 же. Так понятнее?

Представь, что у тебя вероятность выпадения трёх орлов из трёх равна 1/2. Вероятность выпадения трёх решек из трёх тогда тоже равна 1/2, ибо без разницы. Тогда чему же равна вероятность выпадения двух орлов и одной решки, если у нас и так уже в сумме единица набралась?
>> No.68694 Reply
>>68693
Но ведь может быть еще ООР или РОР и тому подобное.
>> No.68719 Reply
>>68691
потому что 1/21/21/2=1/8
>> No.68731 Reply
>>68694
Ну как бы и я про то же ...
>> No.68823 Reply
>>68555
Нужно проверить, что преобразование, переводящее (x,y) в (y,x), сохраняет меру (являющуюся совместным распределением (X,Y)). Для этого достаточно доказать, что оно сохраняет меру прямоугольников. Мера прямоугольника [a,b]x[c,d] равна (F(b)-F(a))(F(d)-F(c)), а мера его образа [c,d]x[a,b] равна (F(d)-F(c))(F(b)-F(a)), где F - функция распределения любой из упомянутых величин, что и требовалось доказать. Из этого сразу следует равенство вероятностей попадания в любую пару областей, отличающихся перестановкой координат. Теорема Фубини тут ни при чём, всё делается на пальцах. Хотя, да, теорию меры знать все равно нужно.
>> No.68845 Reply
File: Photo-14.07.12,-1...
Jpg, 213.35 KB, 720×960
edit Find source with google Find source with iqdb
Photo-14.07.12,-14-38-14.jpg
File: ca11e989726024c6e...
Png, 0.90 KB, 161×57
edit Find source with google Find source with iqdb
ca11e989726024c6e7a8a2ea2f9cb406.png

Доброматемач, привет!
У меня опять ПРОТИВОРЕЧИЕ.
Равен ли k-й начальный момент абсолютно непрерывной случайной величины матожиданию ее k-й степени?
Вики предлагает пикрелейтед №1 для его вычисления.
Но если попытаться вычислить матожидание по определению (пикрелейтед № 2, на примере плотности f(x) = 2х и второго момента) то получится разница в интегральных скобках, и соответственно разные значения интегралов. ЧЯДНТ?
>> No.68847 Reply
>>68845
Борда переставила пикрелейтеды местами, вот ведь!
Короче, тут их надо переставить местами, чтобы они соответствовали отсылкам в посте.
>> No.68891 Reply
>>68845
Разобрался. Знал бы сразу, что при замене х на х(t) надо домножать на производную обратной функции - вообще бы молодец был.
>> No.68902 Reply
File: 1252333347314.jpg
Jpg, 58.24 KB, 440×500
Your censorship settings forbid this file.
unrated
Анон, а бывают на прямой несчетные семейства непересекающихся неизмеримых по Лебегу множеств?
>> No.68904 Reply
File: 01030.png
Png, 382.10 KB, 640×360 - Click the image to expand
edit Find source with google Find source with iqdb
01030.png
>>68902
> несчетные семейства непересекающихся неизмеримых по Лебегу множеств?
> несчетные семейства
> неизмеримых по Лебегу множеств
Ты себе как это представляешь? Неизмеримые - они на то и неизмеримые, что их меру вычислить невозможно. Соответственно, говорить о мощности семейства множеств, у которых невозможно вычислить меру - в общем, у меня даже не хватает воображения на умное сравнение.
>> No.68905 Reply
>>68904
Дык их лебегову меру нельзя, а вот "количество" семейств прикинуть можно.
>> No.68906 Reply
>>68904
Ты путаешь меру и мощность. И хватить аватаркофажить няшкой из Tatami Galaxy, это я ей аватаркофажу :3
>>68905
Множество Витали ведь несчётно, поэтому вряд ли ты такое семейство более чем из счётного количества найдёшь.
>> No.68913 Reply
>>68904
Чего-то ты совсем не в теме, няша. У любого множества есть мощность. Известен такой факт?
>>68906
Поскольку декартов квадрат бесконечного множества ему самому равномощен, несчетное количество непересекающихся несчетных подмножеств бывает. В качестве упражнения можете найти в канторовом множестве на [0,1] континуум непересекающихся множеств, гомеоморфных канторову множеству. Но они, к сожалению, все будут измеримы, и поэтому даже если брать канторов дисконтинуум положительной меры, то среди таких подмножеств будет лишь не более чем счетное число подмножеств положительной меры, и ничего не выйдет.
>> No.68924 Reply
>>68913
> У любого множества есть мощность.
Пруф?
>> No.68949 Reply
>>68924
Казимиров Н. И. "Введение в аксиоматическую теорию множеств", теорема 3.4.
Этот факт, кстати, эквивалентен аксиоме выбора :3
>> No.69025 Reply
Анон, не подскажешь хорошей литературы по структурам и алгоритмам обработки данных? Особенно интересует теория графов. Что-нибудь не очень объёмное и легко-читаемое, по возможности.
>> No.69069 Reply
Матемач, я опять с задачей по терверу, точнее уже по статистике.
Есть одна генеральная совокупность, даже зачем-то задан ее объем, допустим он равен N.
Известны ее среднее M и отклонение S.
Чему равна вероятность того, что в выборке из n среднее окажется меньше некоторого заданного значения m (m != M)?
>> No.69115 Reply
>>69069
Из-за твоих постов, анон, я тоже решил вспомнить теорвер. И на эту твою задачу уже убил порядочно времени, но так у меня ничего и не получилось. Так что, если ты её всё-таки решишь, запость, пожалуйста, решение. Я теперь не смогу жить спокойно, пока его не узнаю.
>> No.69130 Reply
Котоны, расскажите, что меня ожидает на первом курсе по направлению "Математика"? Какую годную литературу следует почитать для подготовки?
>> No.69142 Reply
>>69069
Строго говоря, даже при n=1 столь скудных данных не достаточно для решения. Несложно сочинить две совокупности, имеющие одинаковые N, M и S, но разные ответы для n=1 и некоторых m.

Или что у нас происходит в задаче вообще? Может быть, задачу нужно читать так, что у нас просто есть N нормальных (M,S) случайных величин, и вероятность считается как в плане какие значения они выберут, так и в плане какие n из них мы будем рассматривать?
>> No.69189 Reply
Доброанон, а почему производная дифференцируемой на (a,b) функции не может иметь разрывов первого рода?
Мне говорят, что это противоречит теореме Лагранжа о среднем. Но вот как именно, блджад, противоречит я никак понять не могу.
Вот вроде на пальцах видно, что если бы был разрыв первого рода - график функции имел мы в этой точке угол, а угол - это вроде как явный признак недифференцируемости. Но как грамотно это доказать ума не приложу.
>> No.69208 Reply
>>69189
Это называется теорема Дарбу: Производная принимает промежуточные значения, даже не будучи непрерывной.

Доказывается так. Пусть F дифференцируема на [a,b], F'(a)=A, F'(b)=B, и, например, A<C<B. Возьмем функцию G(x)=F(x)-Cx. Это непрерывная функция, по теореме Вейерштрасса у неё есть точка глобального минимума на [a,b], обозначим её "c". Поскольку G'(a)<0 и G'(b)>0, точка c не может совпадать ни с a, ни с b (ведь в их окрестностях функция G ещё меньше чем в них). Значит, точка c лежит где-то внутри (a,b). Следовательно, G'(c)=F'(c)-C=0, т.е. F'(c)=C, что и требовалось. Случай B<A аналогичен.

Причём тут формула Лагранжа - не знаю.
>> No.69217 Reply
File: производные.jpg
Jpg, 84.83 KB, 532×274 - Click the image to expand
edit Find source with google Find source with iqdb
производные.jpg
>>69189
Может быть так:
Дифференцируемость->существование производной->существование и равенство правой и левой->нет разрыва первого рода.
>> No.69223 Reply
>>69130
Если так не терпится начать, можешь погуглить лучшие учебники по матанализу, аналитической геометрии, линейной алгебре, дискретному анализу.
Учись программировать и/или пользоваться матпакетами, выше по треду их обсуждали. Тот же матлаб можно скачать с рутрекера. У нас на прикладной математике и информатике семестр матлаба был аж на четвёртом курсе, хотя он бы и на первом ещё как помог. С другой стороны, не знаю, может быть это полезно, учиться без матпакета, мыслить абстрактно.
А вообще лучше закончить с синим дипломом и красной рожей, чем наоборот. Главное не учёба, ещё не известно, пригодится или нет. Главное - отношение с людьми, друзья, девушка, вот без этого в жизни долго не протянешь. Не зацикливайся на математике.
>> No.69224 Reply
>>69223
> А вообще лучше закончить с синим дипломом и красной рожей, чем наоборот.
Лол, всерьёз же воспримет.
> Главное - отношение с людьми, друзья, девушка, вот без этого в жизни долго не протянешь.
Девушка? На мат. специальности? Уж лучше с синей рожей.
Да и друзья, кхм... Это одногруппники, не факт что попадутся годные друзья.
>> No.69225 Reply
>>69224
> Девушка? На мат. специальности? Уж лучше с синей рожей.
Тут уж как повезёт.
> Да и друзья, кхм... Это одногруппники, не факт что попадутся годные друзья.
Одногрупники, не одногрупники - не важно. Главное не быть одному.
>>69130
И ещё, анон. Если ты не выучишь линейную алгебру на первом курсе, то потом будешь жалеть об этом всю оставшуюся учёбу, если не всю оставшуюся жизнь.
>> No.69277 Reply
File: 250 добра.jpg
Jpg, 180.89 KB, 880×400 - Click the image to expand
edit Find source with google Find source with iqdb
250 добра.jpg
>>69223
Спасибо за ответ.
На математике особо зацикливаться не собираюсь, буду стараться делать упор в программирование, благо проблем с самостоятельным изучением нет, что нельзя сказать о математических дисциплинах.
>> No.69290 Reply
>>69217
Заметь, что пределы производной справа и слева вообще существовать не обязаны.
Это не то же самое, что правые и левые производные.
> существование и равенство правой и левой->нет разрыва первого рода.
То есть как ты сделал эту импликацию?
>> No.69302 Reply
>>69290
Я такой мудак, мне так стыдно... Даже контрпример знаю. f(x)=0, при x∈Q; f(x)=x^2, при x∈R\Q. В нуле левые и правые производные есть и равны нулю, а предела производной нет, потому что её вообще нет нигде кроме нуля.
>> No.69331 Reply
File: 7647663595d2b262e0366b2f96018741.jpg
Jpg, 83.30 KB, 347×534 - Click the image to expand
edit Find source with google Find source with iqdb
7647663595d2b262e0366b2f96018741.jpg
>>69302
А вот какая-нибудь f(x)=x^2sin(1/x^2) (где f(0)=0) имеет производную везде, а тем не менее в окрестности нуля производная неограниченна, то есть никаких правых и левых пределов у нее нет.

Так, хорошо, а теперь подумаем. Если у нас функция всюду дифференцируема, то у нас по формуле Лагранжа (f(x+h)-f(x))/h=f'(x(h)) для некой точки x(h) из интервала (x,x+h). А левая часть стремится к f'(x) при h->+0. Поэтому f' имеет в точке x частичный предел, равный f'(x). Отсюда следует, что либо она вообще имеет предел, равный f'(x) (это когда других частичных пределов нет), либо не имеет предела вовсе (когда они есть, хотя бы один).

Так что, пожалуй, да, действительно можно формулой Лагранжа это утверждение получить, хотя до теоремы Дарбу и не дотягивает.

Почему я один отвечаю? Тополог-кун, на кого ты меня бросил?
>> No.69483 Reply
File: rbf_o_GIFSoup.com.gif
Gif, 2335.87 KB, 320×266 - Click the image to expand
edit Find source with google Find source with iqdb
rbf_o_GIFSoup.com.gif
Доброе утро, матемач. Вроде отошел после экзамена, результатов пока не знаю, но кажется кое-что сделать удалось.
Дорогой матемач, спасибо, что помогал готовиться все это время. Спасибо за внимание к моим проблемам, сейчас попробуем их закрыть.
>>69115
>>69142
Аноны, похоже я запостил какую-то дурацкую задачу, возможно ее автор немного упорот. Указаний к решению ее немного, разве что он предлагает использовать ЦПТ. Разве она здесь применима вообще? Знаю человека, который ее вроде бы порешал, обязательно с ним проконсультируюсь.
>>69331
Я кажется понел! Там получается, что если есть конечный односторонний предел у производной, то он обязательно должен быть равен односторонней производной в предельной точке в силу той самой формулы Лагранжа. Но раз у нас по условию функция дифференцируема, то у нее уже есть в этой точке обычная производная. А значит левая производная должна быть равна правой. Но это не так, ведь у нас разрыв первого рода, а в силу той же формулы Лагранжа односторонняя производная со второго конца тоже обязана равняться одностороннему пределу производной. Ура, противоречие!
И подводя итог - если у производной есть разрыв первого рода, то функция обязана быть в точке разрыва как минимум недифференцируемой, как например |x| в нуле. Хух, ну вроде теперь усвоилось.
>> No.69625 Reply
Матемач, а чему равно кардинальное число R^inf?
>> No.69641 Reply
>>69625
inf есть что-то счётное? Тогда континуум. ({0,1}^N)^inf закодировать элементами {0,1}^N легко: первое "число" ставим на нечётные разряды, второе - на чётные не делящиеся на четыре, третье - на делящиеся на четыре но не на восемь, и т.п., на всех хватит.
>> No.69647 Reply
>>69331
> Почему я один отвечаю?
Я болею. Как-то нет сил ответить и чем-то заняться.
> Тополог-кун
Если только диванный.
>> No.69656 Reply
>>69647
А я уж думал я тебя разочаровал ... Ну тогда выздоравливай!

Детская загадка, вдруг кого порадует: описать все функции f:[0,1]->R, стремящиеся к нулю во всех точках отрезка [0,1].
>> No.69698 Reply
Анон, я осознаю, что возможно не по адресу, но не попадалось ли тебе где-нибудь описание реляционной алгебры нормальным математическим языком?
>> No.69699 Reply
>>69656
f(x)=0
>> No.69700 Reply
File: AxGiVU0CEAEML-T.jpg
Jpg, 32.32 KB, 600×450 - Click the image to expand
edit Find source with google Find source with iqdb
AxGiVU0CEAEML-T.jpg
>>69699
А как же функция Римана?
Пока дошел только вот докуда:Если функция непрерывна, то это очевидно тождественный ноль. Разрывы первого и второго рода очевидно недопустимы. Любая функция с конечным числом точек устранимого разрыва подходит. А вот как быть со счетным и несчетным их числом пока не понимат.
>> No.69707 Reply
>>69656
>>69700
По поводу бесконечного числа точек разрыва:
Делим отрезок напополам. По крайней мере в одной половине число точек разрыва бесконечно. Выбираем её. Повторяем ещё раз до бесконечности. Получается последовательность отрезков, на которых f(x)≠0 хотя бы в одной точке. Таким образом края получающихся отрезков сходятся к точке, предел в которой не равен нулю, если он вообще есть. То есть функции с бесконечным числом точек разрыва не подходят.
Таким образом, ответ:
Множество всех функций f:[0,1]->R равных нулю везде, кроме конечного числа точек устранимого разрыва.
Можно конечно и получше сформулировать, но лень.
>> No.69709 Reply
File: rsz_1screenshot-zorich_va_matematicheskij_analiz_t.png
Png, 78.44 KB, 787×515 - Click the image to expand
edit Find source with google Find source with iqdb
rsz_1screenshot-zorich_va_matematicheskij_analiz_t.png
>>69707
Не понимат, все-таки, щто будем делать с пикрелейтед?
Я был неправ даже когда говорил про отсутствие разрывов первого рода, вот это неожиданный поворот.
>> No.69712 Reply
>>69709
>>69707
> Получается последовательность отрезков, на которых f(x)≠0 хотя бы в одной точке.
Да, вот здесь я ошибся. f(x)≠0 не достаточно, нужно чтобы f(x) было больше какого-то ε в любой окрестности.
>> No.69744 Reply
>>69709
f(1)=1≠0
>> No.69757 Reply
>>66948
Товарищи математики, такой вопрос, допустим есть задача "из жизни", которая принадлежит к неустановленной области математики. Каков алгоритм поиска решения?
>> No.69761 Reply
>>69757
1)понять условие
2)решать, пока не решишь
>> No.69769 Reply
Любое непустое множество натуральных чисел разрешимо тогда и только тогда, когда оно является множеством значений некоторой неубывающей всюду определенной вычислимой функции с натуральными аргументами и значениями.
f(1)=1, а f(n)=2, для любого n>1. Функция вычислима, тотальна, не убывает, задаёт множество {1,2}, но не знаю, как построить алгоритм, определяющий по этой функции, принадлежит ли 3 этому множеству или нет. Как быть?
>> No.69773 Reply
>>69769
Типа так? f(1)+f(n)=3, n>1
>> No.69774 Reply
>>69773
Не понял. Можно ведь выполнить лишь конечное число вычислений функции.
>> No.69784 Reply
>>69761
Как решать, если, во-первых, не знаешь соответствующий раздел и, во-вторых, не знаешь какой это раздел?
>> No.69785 Reply
>>69784
Что за задача?
>> No.69787 Reply
Сейчас готовлюсь к ЕГЭ по математике в следующем году.
Вроде бы всё хорошо, но очень не нравится тригонометрия. Точнее мне кошерно решать уравнения, системы их, брать производные и интегралы, действовать с комплексными числами. С геометрией также. А вот всякие преобразования тождественных тригонометрических выражений люто, бешено раздражают. >:3
В дальнейшем, после школьной программы, функции sin(x), cos(x), tg(x), ctg(x), а точнее — нужда в сложных преобразованиях выражений, их содержащих; — будет? Или только в качестве единичных примеров в области интегрирования функций, что я наблюдаю сейчас? Можно будет после ЕГЭ забыть о тригонометрии?
>> No.69789 Reply
>>69769
Реквест отменен. Доказательство не обязано быть конструктивным (можно использовать закон исключенного третьего).
>> No.69791 Reply
File: zorich.jpg
Jpg, 175.38 KB, 843×588 - Click the image to expand
edit Find source with google Find source with iqdb
zorich.jpg
>>69709
В Зориче устранимые разрывы считаются первого рода. Все разрывы функции Римана устранимые, все разрывы в нашей задаче, очевидно, устранимые.

>>69744
Ну и что? Задачка - специально чтобы студент вспомнил, что в определении предела фигурирует именно "проколотая" окрестность!

Подсказки:
Да, ничего принципиально хуже, чем функция Римана, не возникает.
Пользуйтесь леммой Гейне-Бореля (компактностью отрезка).
>> No.69806 Reply
>>69787
Будет. Например, при интегрировании.

Для тригонометрических формул достаточно помнить пару основных, а остальные легко из них выводятся.
>> No.69811 Reply
>>69789
Откуда эта задача?
>> No.69821 Reply
>>69785
Какая разница? Вопрос — общий.
Сгладить график, например.
>> No.69829 Reply
>>69821
> Какая разница?
Да просто странные вопросы. Если задача из жизни, то соответствующий раздел наверняка есть.
>>69784
> Как решать, если, во-первых, не знаешь соответствующий раздел?
Нужно сначала разобраться в соответствующем разделе. Но если задача простая, и хватает знаний из известных тебе разделов, то можно не разбираться.
> Как решать, если не знаешь какой это раздел?
Сначала узнать, что это за раздел. Задача творческая. Можешь на форуме спросить, можешь погуглить, можешь спросить у преподов в своём вузе, жалко сейчас лето, можешь ещё что-нибудь придумать.
Спасибо Капитану Очевидности за помощь с ответами! Без него я бы не справился!
> Сгладить график, например.
Я, конечно не шарю, но звучит так как будто это очень распространённая и много раз решённая задача.
Что ты подразумеваешь под сгладить? Чем задан график? Функцией или точками? Если функцией, то тебе нужно какое-нибудь приближение, хоть многочленами, с минимумом среднего отклонения и максимума второй производной. Как-нибудь так.
Если график задан точками то просто гугли алгоритмы сглаживания кривых.
Если ты научную работу пишешь, то это сложно, а если так просто за чем-то понадобилось, то всё просто и любое решение подойдёт.
>> No.69837 Reply
>>69784
Чтобы знать какой раздел, нужно просто иметь образование; оно, грубо говоря, только для этого и даётся - особо популярные разделы изучаешь в лоб, а про другие просто слышал краем уха, и когда надо - название вспомнится. Если соответствующего образования нет - нужно спросить того, у кого есть. Вроде так примерно получается ...
>> No.69865 Reply
File: IMO_logo.png
Png, 15.29 KB, 220×151 - Click the image to expand
edit Find source with google Find source with iqdb
IMO_logo.png
Добрач, объясни почему лучшие результаты на Международной олимпиаде школьников по математике показывают преимущественно азиаты? Алсо, что знаешь о их подготовке?
>> No.69867 Reply
>>69829
>>69837
То есть без образования и/или помощи посторонних никак?
Тогда такой вопрос: как проще всего научиться соответствующему навыку самостоятельно? Выучить несколько популярных разделов? Как вообще люди без бэкграунда занимаются математикой самостоятельно? Спасибо.
>> No.69888 Reply
>>69865
> по математике показывают преимущественно азиаты
Потому что у них IQ в среднем ~110, у белых ~100, а у ниггеров ~90.
Алсо некоторые азиаты — китайцы, если мы говорим о международном уровне. Там очень много народа ведь, не так ли? И поэтому overдохуя конкуренция. Также. Когда ты спишь — они грызут матан, когда ты доброчуешь капчу — они коммунючуют матан. Когда ты встречаешься с тнями — они встречаются с матаном.
>> No.69892 Reply
>>69865
> показывают преимущественно азиаты?
Потому что мат.олимпиадство, как и любой другой вид спорта требует немереного задротства, а азиаты к этому предрасположены вследствие воспитания и общекультурного фона.
>> No.69895 Reply
>>69865
По Евроньюз есть передача "Мир обучения" или как-то так, там был сюжет про якобы невиданные успехи китайских школьников/студентов в изучении математики. Эти успехи объясняли тем, что китайцы из-за своих иероглифов воспринимают числа слегка не так, как белые люди.
>> No.69901 Reply
>>69895
> воспринимают числа
> математика
У них там счёт или таки логика?
>> No.69909 Reply
>>69901
Доброчую, в математике всего четыре числа - ноль, единица, пипополам и плюсбесконечность. Всё остальное есть только в арифметике.
>> No.69933 Reply
>>69909
А как же ЭКСПОНЕНТА?
>> No.69941 Reply
>>69933
А она выражается через ряд, то есть можно единицами выразить.
>> No.69944 Reply
File: 2011_rus.zip
Zip, 0.27 KB, 0 files - Click the image to get file
view
2011_rus.zip
>>69865
Вот олимпиадные задания. Предлагаю их решить.
>> No.69954 Reply
>>69867
Знаешь что думаю?.. Загляни на какой-нибудь более крупный матемач, типа там dxdy.ru или что-нибудь такое. Некоторое время там повертись, в основном разделе там время от времени появляются хорошие реальные задачи. Ну я там давно не был, но вроде он пока что торт. Так можно немного нахвататься кругозора.
>> No.69956 Reply
>>69867
Если нужно найти подходящий раздел есть ещё вот:
ruwiki://Разделы_математики
Но учить математику самому мне представляется полной жопой.
>> No.69962 Reply
Привет, добрач. Попрошу помощи.
Первого сентября отправлюсь в одиннадцатый класс. Однако без летней подготовки я, скорее всего, не справлюсь с математикой, так как уровень высокий. Но без поддержки не получается даже сесть за примеры, в итоге я уже потерял большую часть лета.
Возникают вопросы, которые сам не могу решить. Не засорять же тред ими.
Я хочу попросить отзывчивого анона общаться по этому поводу в асечке/жаббере. Надеюсь на долговременное общение по теме поста.
Если есть другие идеи, пожалуйста предлагайте.
Не хочется быть в этой область профаном.
Unichi@jabber.ru
>> No.69979 Reply
>>69867
так скажи же наконец, что у тебя за задача!
Я, например, попробую подсказать, какие разделы учить и какие книжки читать.

Алсо, готов обсудить или порекомендовать с любыми желающими литературу по алгебре, геометриям, топологии и т.д.
>> No.69990 Reply
>>69941
Пипополам тогда тоже не нужно. Как, впрочем, и бесконечность.
Только единицы и нули! Хотя, можно обойтись множествами.
Числа не нужны.
>> No.69991 Reply
>>69962
> одиннадцатый класс
> уровень высокий
што
>> No.69995 Reply
>>69990
> Хотя, можно обойтись множествами.
Только эти множества в конце концов как раз случайно окажутся теми самыми числами. И выяснится, что ты всю жизнь говорил прозой, хотя об этом и не знал :3
>> No.69998 Reply
Спасибо, что откликнулись. Добра вам. :3

>>69991
Ничего особенного, с языка сорвалось.

>>69962 - кун
>> No.69999 Reply
>>69962
Матшкола? Если да — то ты сам таки справишься. Если нет — то у тебя там хрень будет, а не математика, которой для решения B/C хватит. В следующем году будем с тобой сдавать. >:3
Я в ССУЗ, но на парах высшей математики играл в покер с посонами на парте задней. А в конце концов сдал экзамен на "отлично".
Проходили комплексные числа, первообразные и интегралы (только неопределённые и определённые), линейную алгебру.
>> No.70007 Reply
>>69944
> Вот олимпиадные задания.
Йопт, это какой курс э-э.. среднеобычного ВУЗа?

Алсо реквестирую годное чтиво по построению матмоделей.
>> No.70042 Reply
>>70007
> это какой курс
А что там есть такое, чего нет в школьной программе? "Множество"? "Функция"? Вроде эти понятия классе в седьмом на интуитивном уровне поясняются (школьник должен знать, что линейная функция - это функция, то есть "ну такой закон, по которому по одному числу вычисляется другое"; он также должен понимать, что геометрическое место точек, про которое говорят во всяких там задачах на построение, или, скажем, решение уравнения или неравенства - это множество, то есть "ну такой набор [совокупность, семейство, стадо / нужное подчеркнуть, ненужное зачеркнуть] точек"), и больше к этому никто никогда не возвращается.

А как решать задачки - это тоже никакой не курс. Хотя, конечно, сравнимо умная студентота по специальности справится получше. Но преимущество будет не в знании определений, а во владении методами.
>> No.70058 Reply
File: 134322681004632.jpg
Jpg, 73.23 KB, 496×480 - Click the image to expand
edit Find source with google Find source with iqdb
134322681004632.jpg
Анон, объясни мне, пожалуйста, что такое производная по Фреше для функции из евклидова пространства размерности n в пространство размерности 1 и приведи пару примеров, пожалуйста. У меня не получается соотнести абстрактное определение с тем, что я уже знаю про анализ функций нескольких переменных.
>> No.70065 Reply
>>70058
> из евклидова пространства размерности n
Ващето производную Фреше придумали для бесконечномерных банаховых пространств.
В конечномерном случае дифференциал по Фреше это просто "обычный" дифференциал.
>> No.70068 Reply
>>70065
Вот мне и нужно понять, каким образом производная по Фреше переходит в обычный дифференциал.
>> No.70071 Reply
>>70068
Пардон, в обычную производную. У меня уже дифференциалы в производные переходят, какой ужас.
>> No.70075 Reply
>>70068
Она в него не переходит, она им является по определению.
В бесконечномерном случае добавляется только одно слово: требуется, чтобы оператор был ограниченным, что автоматически выполнено в конечномерных пространствах.

Асло мне кажется, что ты не совсем твёрдо понимаешь определение дифференциала.
Вопросы для проверки:
1. Дай самое общее известное тебе определение дифференциала, не заглядывая в учебничек
2. В чём различие производной и дифференциала в общем (не одномерном) случае и как они связаны?
>> No.70076 Reply
>>70071
Вот зря ты схлопнулся.
Производная Фреше - это именно таки дифференциал!

Давай начнём с того, что градиент - это не вектор. Это функционал же. Ко-вектор. Хотя у него тоже n координат, они по-другому меняются при замене базиса. Функционал - это такой оператор, только в числа (рассматриваемые как отдельное пространство). dx, dy, dz, ... - это такие вполне конкретные функционалы тоже, dx(xi+yj+zk+...)=x, они образуют базис в сопряженном пространстве - пространстве функционалов. Вот наш градиент, он же производная Фреше, он же дифференциал по этому базису так и разлагается: df=(∂f/∂x)dx+(∂f/∂y)dy+...

Примерно складывается картина? Или в чём-то ещё вопрос?
>> No.70077 Reply
>>70075
> Дай самое общее известное тебе определение дифференциала, не заглядывая в учебничек
Линейная часть производной?
>> No.70081 Reply
>>70075
> > не переходит, она им является по определению.
Не придирайся к словам, я именно это и имел ввиду, что не понимаю, как соотносятся эти два определения.
1. Ну, если говорить про функцию из n-мерного евклидова пространства в одномерное, то дифференциал функции - это разность f(x+h)-f(x) при норме вектора h стремящейся к нулю.
2. Различие в определениях. Частная производная по g - это отношение дифференциала функции к стремящемуся к нулю приращению g. Ну, есть еще условие дифференцируемость, иначе говоря, такое отношение должно существовать и однозначно определятся.
>>70077
не-а. Я даже не особо понимаю, зачем ты градиент приплел.
>>70077
ух ты, сколько народу понабежало.
>>70068-кун
>> No.70082 Reply
>>70081
> > условие дифференцируемости
самофикс.
>> No.70083 Reply
>>70077
> > Дай самое общее
> Линейная часть производной?
И сразу file. В этом месте придётся поработать, если есть желание что-то понять.

>>70076
Простите, милейший, но это:
> градиент - это не вектор
дезинформация.
Кстати, откуда такая любовь к координатам?
А тензоры, например, это такие наборы чисел, которые при замене базиса изменяются определённым образом, да?
Не пытаясь тебя троллировать задеть, предположу, что по-другому вас просто не учат.
В таком случае стоит иметь ввиду, что координатные определения самые хуёвые -- далеко не самый понятный подход.
>> No.70084 Reply
>>70081
> дифференциал функции - это разность f(x+h)-f(x) при норме вектора h стремящейся к нулю.
Дык это ж ноль для любой непрерывной функции.
Низачот, не.
>> No.70085 Reply
>>70084
Хм, ну если я буду очень строго рассуждать, то получается выражение df=f(x+h)-f(x)+o(|h|), причем o должна иметь более высокий порядок малости, чем h->0.
>> No.70086 Reply
>>70081
> дифференциал функции - это разность f(x+h)-f(x)
нет, НЕТ. Читать английскую вики и второй том Зорича, глава 2.
Быстро решительно!
Какие уж тут фреше.

2. Если коротко, то дифференциал -- отображение касательных пространств, а производная -- отображение, которое каждой точке ставит дифференциал.
>> No.70087 Reply
>>70085
До конца - если это выражение существует, то оно и называется дифференциалом функции f.
>> No.70088 Reply
>>70087
формально больше похоже на правду, но ты всё равно не понимаешь сути
там должно быть df применённое к касательному вектору h.

>>70086>>70083-кун
>> No.70089 Reply
>>70083
Какая разница координатное определение или не координатное? Тензор - это вполне конкретная вещь, пусть и много определений, но эквивалентных. Тип тензора - тоже. Либо мы пишем, что градиент есть (∂f/∂x^i), и тогда это не вектор а нечто ковариантное, либо отдельно поднимаем индексы, но таких определений я нигде не видел.
>> No.70090 Reply
>>70088
может, объяснишь тогда? Пожалуйста.
>> No.70092 Reply
>>70086
А что такое касательные пространства?
Нубо70077
>> No.70093 Reply
>>70089
> Какая разница координатное определение или не координатное?
разница в том, что одни дают понимание, а другие нет.
> много определений, но эквивалентных
ещё бы они были не эквиваленты! не философию же обсуждаем всё-таки

По добробратски подскажу тебе самое простое безкоординатное определение градиента функции.
Градиент функции в точке -- это такой вектор, что скалярное произведение с ним равно производной функции по данному направлению.

>>70090
Ох, для этого придётся написать небольшую простыню текста.
>> No.70094 Reply
>>70093
бескоординатное, фикс, лол
>> No.70097 Reply
>>70093
> > Ох, для этого придётся написать небольшую простыню текста.
Если тебя не затруднит.
>> No.70101 Reply
>>70093
> скалярное произведение
Ну няша, ну пока ты живёшь на таком уровне - конечно ты не чувствуешь разницы между вектором и ковектором. Теперь тебе осталось признать, что твое бескоординатное определение не эквивалентно тому координатному.
>> No.70102 Reply
>>70097
Ну ок, я сжато напишу в общих чертах. За подробностями настоятельно рекомендую
обращаться ко второму тому Зорича.
Итак.
1. Между множествами бывают отображения. Некоторые множества имеют такую структуру,
что для них можно определить касательные пространства, только в этом случае можно
говорить о дифференциале отображения.
2. Тривиальные примеры касательных пространств всем известны: для кривой на плоскости
это соответствующая прямая, для 2мерной поверхности в R^3 это соответвтующая плоскость
(напоминать их определения, думаю нет смысла). Для R^n касательным пространством является
само же R^n, только начало координат находится в интересующей нас точке. Это немного тонкий
момент, который нужно понимать.
Кроме того, касательные пространства можно легко определить и для многообразий (это обобщение
"поверхностей"), которые вообще никуда не вложены, для алгебраических многообразий и т.д.
Например, для абстрактных многообразий, касательные векторы можно понимать как классы эквивалентности
параметризованных путей, проходящих через данную точку, пути эквивалентны, если в карте (любой, не важно)
они имеют один и тот же вектор скорости. Т.е. на многообразии они "касаются в этой точке". Это же
определение эквивалентно тому, что касательные векторы -- дифференцирования ростков функций
(если сразу непонтяно, то не страшно это не так важно для понимания сути дифференцирования).
3. Итак, касательные пространства это по определению векторные пространства. А **дифференциал отображения
это всегда линейное отображение касательных пространств**. Но как дифференциал связан с самим отображением?
Чтобы не перегружать рассказ буду писать сразу для отображений R^n в R^m (для многообразий определяется тоже
не сложно). Итак, есть точка "х", мы знаем, что касательное пространство к ней это само R^n с началом координат
в "х", возьмём в нём вектор "h". Этот касат. вектор должен как-то отобразиться в касат. пр-во в точке f(x).
По определению отображение дифференцируемо в точке "х", если для любого такого "h"можно записать
f(x+h)-f(x)= A(h) + o(h,x).
A(h) -- опреатор из касат. пр-ва в "х" в касат пр-во "f(x)".
Этот оператор А и называется дифференциалом в точке "х."
Для того, чтобы найти его запись в ортонорм. координатах базиса касат. пр-ва, который получается параллельным
переносом базиса самого пр-ва, в котором определена ф-ция f, мы будем просто задавать вектор h как смещение по
одной координате и смотреть предел. В итоге получим всем известную матрицу частных производных.

В общем, я не удивлюсь, если текст покажется слегка запутанным. Поэтому ещё раз отсылаю к учебнику Зорича.
>> No.70103 Reply
File: 1281003604073.jpg
Jpg, 37.22 KB, 691×483 - Click the image to expand
edit Find source with google Find source with iqdb
1281003604073.jpg
>>70101
> на таком уровне
а-ха-ха, это на каком -- "таком"?
Я надеюсь, любому известно что скалярное произведение задаёт изоморфизм касательного и кокасательного пространств. Не на этом ли основании ты их путаешь?
Кстати, моё определение, конечно, с очевидностью эквивалентно координатному. А то что ты написал выше неверно без одно уточнения. Так-то.
>> No.70104 Reply
>>70102
важное уточнение
> f(x+h)-f(x)= A(h) + o(h,x).
> A(h) -- опреатор из касат. пр-ва в "х" в касат пр-во "f(x)".
опреатор из касат. пр-ва в "х" в касат пр-во "f(x)" применённый к касат. вектору h
>> No.70105 Reply
Народ, у вас градиент & дифференциал на каком курсе вообще проходят?
А то впечатление будто не самая успевающая первокурсота одна тут. Ну или дураки, закатывающие тексты с несбалансированным отношением смысла к терминологии.
>> No.70106 Reply
>>70105
У меня о них упомянули на первом курсе и больше к этой теме не возвращались.
>> No.70107 Reply
>>70106
Упомянули или именно изучили с доказательством и последующими конструкциями?
>> No.70108 Reply
>>70105
о, в нашей милой беседе ещё один участник.
А что сам мсье думает по поводу градиента? лол
>> No.70113 Reply
>>70108
Градиент f в точке (х, у, z) это векторнозначная функция ∇f(x, y, z) равная <∂f/∂x, ∂f/∂y, ∂f/∂z>. ∂f/∂x и ∂f/∂y - частные производные.
Здесь геометрический смысл градиента в наибольшей скорости изменения функции (нескольких переменных).
>> No.70116 Reply
>>70113
Опять координатное определение и опять не полное.
А на каком курсе учат давать определения не до конца? лол
>> No.70120 Reply
>>70116
Спрашивать, чтоб поспорить - это конечно challenging, но я ограничусь следующим: предложенный здесь "безкоординатный ваирант" а-ля "градиент - это такая штука, которая при скалярном произведении даёт производную по направлению" - вообще трудно счесть за какое-либо определение.
  
Ну и, конечно, определение правильное, взятое из учебника. Другое дело, что для градиента в 3 измерениях. Что для 2, что для n измерений, там будет всё аналогично.
Вот заслуживает внимания то, что в этом треде пока ещё не назвали ахтунгом приравнивание градиента к дифференциалу, что, несомненно, свидетельствует об уровне, высоком и непробиваемом.

>>70076
> они образуют базис в сопряженном пространстве
> df=(∂f/∂x)dx+(∂f/∂y)dy+...
И да, это - дифференциал. Но не градиент, лол.
  
От собралось, блин, "топологов".
>> No.70122 Reply
>>70120
> вообще трудно счесть за какое-либо определение
А что же тебе мешает, не поделишься?
"Неучи" из английской вики (в русскую я не заглядываю), автор классического учебника В.А. Зорич, директор НМУ Ю.С. Ильяшенко считают именно такое определение изначальным.
Но ты не уклоняйся от ответа, а поясни, чем оно тебя смутило.
> взятое из учебника
А из какого учебника? Я знаю много негодных книжек и список постоянно пополняется.
А определение, которое ты дал, не полное, предлагаю поискать неточность. Или указать?
>> No.70124 Reply
>>70122
> а поясни, чем оно тебя смутило.
Ссылаемостью на более сложную конструкцию.
> А определение, которое ты дал, не полное
И дальше что?
>> No.70127 Reply
>>70124
> Ссылаемостью на более сложную конструкцию
Это скалярное произведение более сложная конструкция?
Мсье запутался в трёх аксиомах?
Ну хорошо, скажу прямо, а как мсье определит градиент в неортогональных координатах?
> И дальше что?
Ха-ха, он пришёл в тред с видом знатока, дважды проявил невежество, а теперь говорит: И ДАЛЬШЕ ЧТО?
>> No.70130 Reply
File: cdaa43f1e3edb907dd05599af092033a.png
Png, 2.62 KB, 446×53 - Click the image to expand
edit Find source with google Find source with iqdb
cdaa43f1e3edb907dd05599af092033a.png
>>70127
> Это скалярное произведение более сложная конструкция?
Скалярное произведение градиента и единичного вектора и просто градиент - какая конструкция менее громоздка? А впрочем не отвечай, это риторический вопрос.
> Ну хорошо, скажу прямо, а как мсье определит градиент в неортогональных координатах?
Подразумевается, что это невозможно? Определение градиента в таком случае - пикрилейтед. Выглядит нагромождением, но я и не говорю о неортогональных координатах или на-все-случаи-жизни универсальных случаях.
> Ха-ха, он пришёл в тред с видом знатока, дважды проявил невежество, а теперь говорит: И ДАЛЬШЕ ЧТО?
У меня бабушки с соседнего подъезда менее сварливы.
Тем временем: франкоман-оппонент (а вернее отчаянно старающийся таковым быть) спросил о моём виденьи градиента, я ответил, и намеренно в координатах, ибо в этом треде через частные производные выражали дифференциал и называли его градиентом. И вот в ответ на коррекцию грубейшей ошибки в данном треде я получаю лично мне не нужные заявления о том, что определение неполно - обратное, кстати, и не утверждалось.
Так что я всё же спрошу: what's your point? Я к этому вашему спору о полноте и "что же лучше: координатное/не координатное представление" отношения не имею, это уж на всякий случай.
>> No.70133 Reply
>>70130
> Скалярное произведение градиента и единичного вектора и просто градиент
что за бред? какой ещё единичный вектор? одна история охуительнее другой просто.
Ты уверен, что понял определение, на котором я настаиваю?
> Определение градиента в таком случае - пикрилейтед.
Уродливо очень, конечно, но выкрутился.
А как мсье даст определение градиента функции, заданной на многообразии с римановой метрикой?
Всё-таки я занимаюсь полемикой не только ради развлечения и самоутверждения, но и хотел бы попробовать наводящими вопросами
подвести к пониманию, хотя бы в качестве побочного продукта.
> и намеренно в координатах
да-да-да, я не сомневаюсь в том, что ты постоянно имел ввиду именно ортонормированные координаты, хотя ни разу не упомянул об этом, пока я не указал на это явно.
>> No.70136 Reply
>>70133
Ты сначала разберись, в чём твоя претензия.
> какой ещё единичный вектор?
> что скалярное произведение с ним равно производной функции по данному направлению.
Так-с, производная по направлению - это скалярное произведение градиента функции на единичный вектор, являющийся ортом направления.
Ну вот, теперь я даже не знаю, понял ли ты определение, на котором сам настаивал.
>> No.70137 Reply
>>70136
> в чём твоя претензия
Ты втесался в нашу милую беседу с видом знатока, я решил пощёлкать тебе по носу, и ты вполне ожидаемо проявил невежество. Какие тут претензии?
> производная по направлению - это скалярное произведение градиента функции на единичный вектор, являющийся ортом направления
> производная по направлению - это скалярное произведение градиента функции на единичный вектор, являющийся ортом направления
> производная по направлению - это скалярное произведение градиента функции на единичный вектор, являющийся ортом направления
А вот постепенно-постепенно из тумана и вырисовываются причины непонимания.
Это определение тоже написано в говноучебнике, название которого я так и не увидел?
Ты определяешь производную по направлению через градиент, а градиент через производные по направлению.
В своём ли ты уме?
Ну ничего, я обучу тебя анализу, если, конечно, ты не будешь сильно сопротивляться, лол.
А пока повтори определение производной по направлению.
enwiki://Directional_derivative
>> No.70139 Reply
File: 6ed6c035c91d87ee286655aea317b023.png
Png, 0.69 KB, 173×21 - Click the image to expand
edit Find source with google Find source with iqdb
6ed6c035c91d87ee286655aea317b023.png
>>70137
> Ты втесался в нашу милую беседу с видом знатока, я решил пощёлкать тебе по носу, и ты вполне ожидаемо проявил невежество. Какие тут претензии?
За исключением того случая, что я оказался прав? Это уже не самоутверждение, это аутотренингом называется.
Ну а бесцельную полемику вести я не люблю.


Дальше - я ржал диким хохотом.

Вот тест, не идиот ли ты, местный "тополог".
> Градиент функции в точке -- это такой вектор, что скалярное произведение с ним равно производной функции по данному направлению.
Это твои слова. Так, производной по направлению является скалярное произведение градиента и единичного вектора.
И затем следует отрицание вышеобозначенного и недоумевание, откуда в этой фразе взялись единичные векторы.
Так вот, ты всё отрицаешь и отрицаешь - а с какой целью? Отрицание ради отрицания - явный признак идиотизма. Особенно при зашкаливающем фоне нудности.

Далее про "говноучебник" - это, вообще, из той же ссылки на enwiki.

В общем, хватит мне на сегодня сеанса "да-нет, нет-да", я для таких случаев лучше попугайчика куплю.
>> No.70140 Reply
>>70139
Возможны два варианта:
1. Ты действительно не понимаешь своей глупости (точнее говоря необразованности)
2. Ты принципиально отказываешься её признать.
Оба случая мне интересны: в одном научу, в другом посмеюсь.
====
Я начинаю чувствовать себя Луговским.
Итак, во-первых ты исказил сам факт: производная по направлению равна скалярному произведению градиента
не с ортом направления, а с самим вектором. Производная на направлению линейно зависит от вектора этого самого
направления.
====
Во-вторых в математике есть простой принцип: нельзя определять А через Б, а потом определять Б через А. Ты же определил
градиент через частные производные, а потом производные по направлению (т.е. и частные производные) через градиент.
Поэтому я и спросил, в своём ли ты уме? Это бы не риторический вопрос.
Ты увидел в вики и прикрепил к сообщению формулу, которая и является определением градиента. А я тебя ещё пару постов
назад послал в ту же вики читать определение градиента. Ты это сделал? Сделал?
====
И ещё я всё же хотел бы увидеть название учебника, по которому ты изучал анализ. Почему ты его скрываешь?
>> No.70142 Reply
File: math-st152_1.zip
Zip, 3.39 KB, 0 files - Click the image to get file
view
math-st152_1.zip
>>70140
Первокурсота врывается в тред, потрясая томом Письменного (стр. 504).
grad U = (∂U/∂x) i + (∂U/∂y) j +  (∂U/∂z) k
Градиент - это вектор, координатами которого являются значения частных производных функции U(x; y; z) в точке M(x; y; z).
>> No.70165 Reply
>>70140
> Ты же определил градиент через частные производные, а потом производные по направлению (т.е. и частные производные) через градиент.
> Ты
> Ты определил градиент через частные производные
Нет, ну всё-таки идиот.
> Ты это сделал? Сделал?
А ещё выеживается.

В общем делааа, теперь я окончательно разуверился в тебе.
>> No.70168 Reply
>>70165
Не хочу тебя обидеть, но ты, эээ, немного тупой.
Это в ответ не на последнее сообщение, а на всю совокупность твоих высеров.
(твоя вина в этом частична: математическое образование в РФ претерпевает наихудшие времена)
От нечего делать я взялся было в стиле Луговского ткнуть тебя носом в твоё невежество,
но ты, видимо, не способен понять даже что я имел ввиду. Что ж, на будущее мне наука.
====
В завершение ещё раз рекомендую тебе (хотя это бесполезно) и остальным участникам треда
читать английскую вики, тома Зорича и учебники Рудина.
>> No.70169 Reply
>>70168
> учебники Рудина
У него их что, несколько?
>> No.70171 Reply
>>70169
Principles of Mathematical Analysis -- для малышей
Real and Complex Analysis -- для ребят чуть взрослее
>> No.70257 Reply
Доброаноны, а каковы пререквизитс (не знаю, как это слово перевести на русский) для теории меры и по каким изданиям лучше всего ее учить?
>> No.70270 Reply
>>70257
Преквизитов нет, лол.
Есть серьёзные книжки типа Халмоша "Теория меры", но для первого чтения она может показаться скучноватой.
В упомянутой выше Principles of Mathematical Analysis Рудина подробно рассматривается построение меры
Лебега и интеграла Лебега, можно попробовать начать с неё.
Ещё у &&няшно&&Вербицкого был курс по теории меры оче хорощий, ищи тут: http://verbit.ru/
>> No.70271 Reply
>>70270
проебал разметку, лол
у няшноВербицкого
>> No.70325 Reply
File: 183847.jpg
Jpg, 32.48 KB, 250×500 - Click the image to expand
edit Find source with google Find source with iqdb
183847.jpg
А кто где учится?
>> No.70352 Reply
>>70325
Я преподаю. Я неудачник?
>> No.70353 Reply
>>70352
Смотря где преподаёшь
>> No.70355 Reply
>>70352
Покажешь план лекций? :3
>> No.70358 Reply
>>70352
Где и на кого учился? Где и что преподаешь?
>> No.70377 Reply
>>70352
Ну, наверное. Скорее да, чем нет. Но хуле там, давай, наливай.
>> No.70392 Reply
>>70257
Дьяченко Ульянов норм.
>>70270
> Преквизитов нет, лол.
Ну матан первого курса надо знать всё-таки, теоретическую часть, так сказать, что такое непрерывная функция хотя бы ... а вообще да.
>> No.70395 Reply
>>70392
Что такое сходящийся ряд тоже ...
Семён
>> No.70407 Reply
>>70168
Да нет, всё же ты.
Отрицать, что определение градиента - через частные производные, - это глупо. И оно такое есть что в Рудине, что в Multivariable Calculus по программе MIT.
Один этот факт сводит на нет все попытки "в стиле Луговского" понаучать, а сами возмущения твои на ровном месте и вовсе смешны.
>> No.70409 Reply
>>70407
Может, тебе ещё труды Бурбаки нравятся?
>> No.70415 Reply
>>70409
Нет. А какое отношение имеет сказанное к Бурбаки?
>> No.70419 Reply
>>70407
> что в Рудине
Заглянул в детскую книжку Рудина. Там заранее указано, что мы будем иметь дело только с ортонормальными базисами только в R^n, а потом говорится, что строка из частных производных -- это "so-called gradient". Ну, эээ, да, а как её ещё-то назвать?
> Multivariable Calculus
Это undergraduate курс?
Меня учили тому, что "калькулус" это ругательное слово, независимо от места, где оно читается.
А в Зорича ты избегаешь заглядывать? Градиент там определяется, кажется в конце первого тома.
====
Когда-то (ещё, кажется, в школе) меня и самого учили, что градиент -- это строчка из частных производных. Но это только потому что в школе кроме R^n ничего и не проходят, лол. На этом маленьком примере видно различие между хорошей математикой и не очень хорошей. Если найдутся желающие, напишу пару тезисов по этому поводу.
====
Кстати, судя по предыдущему треду, определение градиента через частные производные можно смело назвать YOBA-градиент, лол. Аббревиатура идеально подходит.

>>70409
Мне Бурбаки нравятся, некоторые книжки. А что?
>> No.70430 Reply
>>70419
> Заглянул в детскую книжку Рудина.
В какую из детских?
> ортонормальными
По-русски ортонормированными.
> Меня учили тому, что "калькулус" это ругательное слово
Ты не Миша, раз. Зорич - это калькулус, два. Этот термин вполне прижился в разделе, излагающем обычную дифф. геометрию через пучки, три.
> различие между хорошей математикой и не очень хорошей.
Чтобы судить о математике, нужно сначала вырасти. Математика используется для решения конкретных задач естествознания. Бессмысленно говорить, что одно определение лучше другого, потому как у них разные цели.
> определение градиента
Ну давай своё абстрактное определение.
>> No.70432 Reply
>>70419
> Меня учили тому, что "калькулус" это ругательное слово, независимо от места, где оно читается.
Неправильно учили. Открываешь "английскую википедию" и смотришь.
enwiki://Mathematical_analysis
Прямо под Real Analysis видим Multivariable Calculus. К этому разделу относятся градиент, частные производные, дифференциал.
Vector Calculus, он же Vector Analysis.
enwiki://Vector_calculus
> Когда-то (ещё, кажется, в школе) меня и самого учили, что градиент -- это строчка из частных производных. Но это только потому что в школе кроме R^n ничего и не проходят, лол. На этом маленьком примере видно различие между хорошей математикой и не очень хорошей. Если найдутся желающие, напишу пару тезисов по этому поводу.
Смысл градиента не в строчках, но это и ежу понятно. Бравировать тем, что у кого-то математика хорошая, а у кого-то плохая - глупо, ибо везде есть свои стандарты, и уж либо математика - либо плохой математик плохо видит что есть что.
Здесь как раз-таки частный случай был показателен в рамках творящегося бреда, где равняли дифференциал градиенту. Показательным было то, что градиент это вектор, пусть даже и из частных производных.
> Кстати, судя по предыдущему треду, определение градиента через частные производные можно смело назвать YOBA-градиент, лол. Аббревиатура идеально подходит.
Можешь назвать йобой сам оператор набла, используемый и в тензорном анализе, если легче станет.
>> No.70436 Reply
>>70430
Значица, абисняю.
Наука называется "Analysis", иногда добавляют "Real" или "Compelx" или ещё что-то в этом духе. Под калькулусом подразумеваются вполне конкретные вещи: этим термином называют книжки/курсы упрощённого анализа для инженеров, экономистов, андеградов и т.д., другими словами калькулус="analysis for dummies".
> Ты не Миша
АЙ РОРУДО! неужели ты думаешь, что только Миша придерживается такого мнения и это он меня учил?
> Математика используется для решения конкретных задач естествознания.
А ФИЛОСОФОВ-то и забыли спросить! (если ты не понял иронии: зачем нужна математика -- это дискуссионный и уже лет триста вызывающий бурлящие срачи вопрос)
> Ну давай своё абстрактное
> своё
Может ты мне ещё и определение производной, которую придумал Ньютон, припишешь? Почитай тред.
====
>>70432
> везде есть свои стандарты
вот-вот
Я говорю об обучении математиков. Как нужно учить градиентам и всему-всему остальному физиков, инженеров, программистов и т.д., мне неизвестно, но я сильно уверен (но не настаиваю), что и их нужно учить хорошей математике.
По поводу калькулуса см. выше.
> Можешь назвать йобой бла бла бла
Я назвал йобой то, что назвал. Ты хочешь что-то додумать за меня?
>> No.70442 Reply
>>70436
> Я говорю об обучении математиков. Как нужно учить градиентам и всему-всему остальному физиков, инженеров, программистов и т.д., мне неизвестно, но я сильно уверен (но не настаиваю), что и их нужно учить хорошей математике. По поводу калькулуса см. выше.
Single-variable, Multivariable (multivariate), а также Vector Calculus (это и есть Vector Analysis!) - must разделы в первом году обучения математиков в любом департменте математики будь то в MIT, Berkeley, Harward или CalTech.
> Я назвал йобой то, что назвал. Ты хочешь что-то додумать за меня?
Набла: <∂/∂x, ∂/dy, ∂/∂z, ..., ∂/∂n>. Она определена, цитирую, in terms of partial derivative operators. И применительно к скалярному полю даёт градиент - так он и выглядит, с частными производными.
Так что если ты признаёшь такое описание градиента ЙОБОЙ, то и наблу можно не игнорировать.
>> No.70446 Reply
>>70442
> первом году обучения математиков
Андеград первокурсники в "будь то в MIT, Berkeley, Harward или CalTech" -- это выпускники американских школ. Они феноменально слабы даже по сравнению с рашкинской школотой. Им и нужен курс калькулуса, чтобы они не путали сложение с умножением и привыкли к декартовой системе координат.
калькулус="analysis for dummies"
> Набла
Йоба -- это путать наблу и градиент. Они совпадают только в R^n и ортонормальной системе координат. Это я пытаюсь донести до тебя последние десять сообщений в этом треде.
>> No.70448 Reply
Аноны, позвольте вам выразить моё восхищение. Читаю ваш тред, нихрина не понимаю, но это всё так загадочно и чудесно! Это как смотреть аниму на лунном со слетевшим драйвером видеокарты.
после 6Х8 таблицу умножения не твёрдо знаю-кун
>> No.70455 Reply
А меня последние сообщения в треде наоборот погружают в пучину отчаяния. По-моему срачу об определениях не место в /u/. Вы бы лучше решили всё в личной переписке, а сюда запостили, чем всё кончилось.
>> No.70459 Reply
>>70455
Нет уж, пусть дальше беседуют. Я вот из их беседы узнал, что у Рудина несколько книжек, и получил наводку на Зорича, а сейчас вот про наблу и даламбертиан что-нибудь новое услышу. Всегда думал, что набла в разных системах координат по-разному записывается, ruwiki://Оператор_набла_в_различных_системах_координат , а тут наблу, кажется, готовятся обобщить.
>> No.70491 Reply
File: 1238f08c69fb4613b...
Png, 1.96 KB, 342×52
edit Find source with google Find source with iqdb
1238f08c69fb4613ba9fdd67fdf4406e.png
File: b3446cd1216fb2f9f...
Png, 2.22 KB, 408×53
edit Find source with google Find source with iqdb
b3446cd1216fb2f9ff61cd87693b40c3.png

>>70446
> Андеград первокурсники в "будь то в MIT, Berkeley, Harward или CalTech" -- это выпускники американских школ. Они феноменально слабы даже по сравнению с рашкинской школотой.
Сюрприз, такая же программа в Англии для выпускников А-levels. И в Европе: во Франции, в Германии - практически всюду.
> калькулус="analysis for dummies"
Калькулюс вообще нельзя равнять широкому раздела анализа - это было бы действительно for dummies. Различные виды Калькулюса: Vector, Multivariate, Stochastic Calculus & Calculus on Variations - это различные разделы мат. анализа.
А то ты как-то игнорируешь что свою любимую "английскую вики", что программы обучения математиков в НЕрашкинских ВУЗах.
> Йоба -- это путать наблу и градиент. Они совпадают только в R^n и ортонормальной системе координат. Это я пытаюсь донести до тебя последние десять сообщений в этом треде.
Вообще-то. Никто их не путает. Формула градиента в неортонормированных координатах уже была. Один пик - по градиенту, один - по дивергенции, и всё это с наблой.
Впрочем, о совпадении речи не шло - не хочешь упоминать наблу и не надо.
>> No.70508 Reply
Я что-то не понимаю. Мне достаточно трудно вникнуть в суть спора, потому что лень читать его с самого начала, но по-моему, если отбросить философию и взаимные оскорбления, то вы спорите об одном-единственном определении. Что за фигня здесь происходит?
>> No.70510 Reply
>>70508
Чад кутежа, очевидно же.
>> No.70512 Reply
>>70508
> Что за фигня здесь происходит?
Градиентный триангулят ортонормированного базисно-интегрального кластера n-мерных массивов хуиты — для нас, быдла; я гарантирую это. :3
>> No.70517 Reply
File: MakeoverIdea.jpg.jpg
Jpg, 33.59 KB, 299×397 - Click the image to expand
edit Find source with google Find source with iqdb
MakeoverIdea.jpg.jpg
Аноны, извините за нубский вопрос, но зачем вообще нужны гомеоморфизмы? Ну вот допустим бублик гомеоморфен собаке, и что это нам дает?
>> No.70528 Reply
>>70491
> в Англии во Франции, в Германии - практически всюду.
А то! И в Раше тоже: то что преподают в быдлоинженерных вузах -- это именно калькулус, а не "матматический анализ". Про Америку я вспомнил, потому что заграничное образование ассоциируется в первую очередь с ней. Я не могу, конечно, строго обосновать, почему "калькулус" это неприличное слово, не теорема же. Это знание просто является общим местом в математическом сообществе.
====
Вернёмся к нашим градиентам. Подведём итог нашей содержательной беседы.
Как ты теперь определишь градиент функции, заданной на многообразии?
Если нужны координаты -- пожалуйста, карта с заведомо криволинейными координатами.
Если нужно скалярное произведение -- пожалуйста, на многообразии задана Риманова метрика.
>>70517
> зачем вообще нужны гомеоморфизмы?
чтобы жидотопологи могли получать гранты, очевидно.
> бублик
> собаке
эээ, а какая дырка у собаки имеется ввиду?
>> No.70529 Reply
в дополнение к >>70528
> то что преподают в быдлоинженерных вузах -- это именно калькулус, а не "матматический анализ".
ну и на мехмате, матмехе, физтехе и т.д. конечно в большинстве случаев тоже. Исключения бывают, но крайне редко, судя по отзывам.
>> No.70532 Reply
>>70528
Няш, несмотря на мой пикрелейтед и обилие искрометных терминологических шуток, я правда хотел бы знать, в чем смысл приложения таких усилий.
>> No.70534 Reply
>>70532
> приложения таких усилий
???
усилий по использованию понятия "гомеоморфизм" или каких усилий?
>> No.70540 Reply
>>70534
По доказыванию, что Х гомеоморфно Y.
>> No.70546 Reply
>>70528
> эээ, а какая дырка у собаки имеется ввиду?
Их не так много.
>> No.70552 Reply
>>70517
Погугли "приложения топологии".
http://www.y10k.ru/books/detail7220.html вот, например.
>> No.70564 Reply
File: img7.png
Png, 0.81 KB, 118×60 - Click the image to expand
edit Find source with google Find source with iqdb
img7.png
>>70528
> А то! И в Раше тоже
Не, в Раше системы чёткой нет. А так - американские школьники в большинстве своём берут наряду с пре-калькулюсом (алгебры-тригонометрии) дифф. и инт. исчисления, что в этой стране и принято называть калькулюсом. Отсюда и растут ноги у стереотипа.
> Это знание просто является общим местом в математическом сообществе.
Не твоя правда. В математическом сообществе знают, что Векторный Калькулюс, Калькулюс многих переменных и так далее - это разделы анализа.
> Как ты теперь определишь градиент функции, заданной на многообразии?
> с заведомо криволинейными координатами.
> задана Риманова метрика.
Это последний реквест, ибо "координатное/безкоординатное", "полное/неполное" - я к этой вашей полемике отношения мало имею.
Однако, допустим и такой вариант - пикрилейтед. Более-менее общий случай, не только для ортонормированных координат.
>> No.70828 Reply
Доброняши, нужно выучить эту вашу математику хотя бы до уровня что дается на технических специальностях в вузах, но желательно выше. Во время ученичества в сих заведениях ебланил на парах высшмата и кое-как сдавал экзамены на троечку, сейчас очень нужно, какой литературой обмазаться.
>> No.70957 Reply
Анон, ко мне тут подошел мой дядя и задал мне весьма странный вопрос, как студенту технической специальности. Вопрос был такой "Почему фигуры одинакового периметра могут ограничивать различную площадь?". Я вообще в ступоре, я даже не представляю, что ему ответить, потому что я тут никакого парадокса не вижу. Вот что ему сказать?
>> No.71010 Reply
>>70957
> Почему фигуры одинакового периметра могут ограничивать различную площадь?
> Вот что ему сказать?
"Потому что автомобили одинаковых цветов ездят с разной скоростью"
>> No.71014 Reply
>>71010
Да-да, я то же самое подумал, когда этот вопрос от него услышал, только про велосипеды.
>> No.71051 Reply
File: Арпрогр.png
Png, 2.09 KB, 400×100 - Click the image to expand
edit Find source with google Find source with iqdb
Арпрогр.png
Как выполнять задания "Докажите что-то там"?
В частности — как доказать, что пикрилэйтед тождество справедливо для любой арифметической прогрессии? Долго втыкал в формулы, но не понял, как вообще с этим работать. Какой тут принцип?
>> No.71057 Reply
>>71051
Наверняка можно по математической индукции
>> No.71067 Reply
File: -.png
Png, 18.30 KB, 595×471 - Click the image to expand
edit Find source with google Find source with iqdb
-.png
>>71051
Читай ruwiki://Математическая_индукция
Подходит для многих задач, особенно где всякие последовательности.
>> No.71068 Reply
>>71051
> Как выполнять задания "Докажите что-то там"?
lol'd
А какие ещё задания могут быть при изучении математики? "Подставьте циферки в алгоритм"?
====
Если дана сумма дробей, то есть стандартный известный трюк: попробовать разложить каждую дробь в сумму так, чтобы все промежуточные посокращались и остались только первая и последняя (иногда такое делается при определении сходимости).
Пусть a(k+1) = ak + b b -- "разность прогрессии" или как там называется эта хуёвина?
Тогда 1/(ak * a(k+1)) = (1/b)*(1/ak - 1/a(k+1))
После этого левая часть твоего равенства превращается в (1/b)*(1/a1 - 1/an). Дальше очевидно.

P.S.
>>70564
То, что на твоей картинке не имеет отношения к градиенту.
>> No.71069 Reply
>>71068
проебал разметку
так где "k" курсивом -- это индекс на самом деле
>> No.71070 Reply
>>71051
> Как выполнять задания "Докажите что-то там"?
Ну а если серьёзно, то есть всего три пути:
1. Последовательное применение "модус поненс": из теоермы А следует утверждение А1, из него А2 из него ... из него An, а это то, что нам нужно.
2. По индукции.
3. От противного.
Всё оче просто!
^___^
>> No.71091 Reply
>>70957
Как математик переспрошу, чему это противоречит.

А дяде ответь "жизнь такая".
>> No.71093 Reply
>>71091
> Как математик
этому треду давно не хватало профессиональных математиков!
А какова область ваших научных интересов?
>> No.71096 Reply
>>70957
Очевидный ответ: А почему бы и нет?
>> No.71097 Reply
Реквестирую годных книжек по computer scince. Интересует большой список затрагивающий заметную часть его областей, что бы можно было выбирать. Пока что из рилейтед книжек читал только Кнутавыборочно. сам в CS почти не шарю, но поняв что абстрактная алгебра мне не очень интересна, решил переориентироваться на более близкую мне область, пока не поздно
>> No.71102 Reply
>>71097
Рекомендую учителя Торвальда, господина Таненбаума, у него серии книг на разные области СS.
>> No.71105 Reply
File: 978-5-94057-321-0...
Jpg, 8.99 KB, 200×333
edit Find source with google Find source with iqdb
978-5-94057-321-0.jpg
File: 978-5-94057-322-7...
Jpg, 7.77 KB, 200×330
edit Find source with google Find source with iqdb
978-5-94057-322-7.jpg
File: 978-5-94057-323-4...
Jpg, 9.42 KB, 200×337
edit Find source with google Find source with iqdb
978-5-94057-323-4.jpg

>>71097
> книжек по computer scince
> абстрактная алгебра мне не очень интересна
Между делом говоря, эти науки гораздо ближе, чем может показаться, ты бы не поспешал с выводами.
Всем, конечно, известно, что криптография и теория кодирования это буквально сателлиты общей алгебры,
а вот мода последнего сезона (и, может быть, скоро это будет мейнстрим) это топологические и
алгеброгеометрические методы анализа данных.
Один из примеров:
http://comptop.stanford.edu/
====
По теме реквеста рьяно рекомендую пикрелейтед, всю серию.
Авторы: Верещагин и Шень.
>> No.71107 Reply
>>70957
Нарисовать ему два прямоугольника.
Один со сторонами 3 и 1, другой со сторонами 2 и 2.
>> No.71112 Reply
>>71102
http://www.pearsonhighered.com/tanenbaum/ меня интересуют более "теоретические" книги. Или это не все его книги, или они более "теоретические" чем кажется по названию?
>>71105
Спасибо за ответ, совсем от алгебры отворачиваться не собираюсь. Я не совсем правильно выразился, насчет "не интересна", просто я решил что не буду делать ее главным приоритетом в изучении. Книжки почитаю.
>> No.71113 Reply
>>71057
Спасибо.
>>71067
Спасибо. И за решение спасибо.
>>71068
Спасибо. Похоже, я неправильно думаю. Привык к -> >Подставьте циферки в алгоритм V_V
>>71070
Спасибо. Удивительно — всего три пути вообще на всю математику?

Математическая индукция. Йопт, так ведь ей можно доказать, что все нечётные числа — простые!
1, 3, 5, 7... и дальше по индукции. 1 — простое число, 3 — простое число. Оба нечётные, разность = 2.
>> No.71114 Reply
>>71113
> Спасибо. Удивительно — всего три пути вообще на всю математику?
Бывает еще перебор всех возможных случаев, если их конечное число конечно. Но обычно это сорт оф первый случай.
> Математическая индукция. Йопт, так ведь ей можно доказать, что все нечётные числа — простые!
1, 3, 5, 7... и дальше по индукции. 1 — простое число, 3 — простое число. Оба нечётные, разность = 2.
Ты видимо не понял суть индукции, допустим база у тебя верна(только 1 обычно не называют простым), но перехода P(n)=>P(n+1) ты не доказал.
>> No.71115 Reply
>>71114
> если их конечное число конечно.
   "если случаев конечное число"
фастфикс
>> No.71116 Reply
>>71113
Ты СОВЕРШЕННО не понимаешь в чем суть мат. индукции.

На примере с простыми нечетными числами тебе надо доказать:
1. нечетное число K - простое
2. Если нечетное число N - простое, то число N+2 - простое
Сможешь это доказать - докажешь, что все нечетные числа, начиная с K - простые
но я думаю, у тебя это не получится
>> No.71120 Reply
>>71113
> Удивительно — всего три пути вообще на всю математику?
На самом деле только один.
Индукция -- это "запрограммированный" modus ponens, а "от противного" это modus ponens прочитанный задом наперёд с навешенными отрицаниями.
Но! Это с учётом того, что сейчас очень высокие требования к строгости доказательства.
В античные и средневековые времена ещё был метод доказательства "смотри на чертёж, пока теоерема не станет тебе очевидна" lol
====
> ведь ей можно доказать, что все нечётные числа — простые!
неправильно доказываешь
надо так:
3 - простое
5 - простое
7 - простео
9 - экспериментальная погрешность
11 - простое
дальше по индукции
Кстати! Единица не является простым по уважительным причинам
>> No.71121 Reply
File: flower2.jpg
Jpg, 93.10 KB, 640×482 - Click the image to expand
edit Find source with google Find source with iqdb
flower2.jpg
>>71113
Но ведь все натуральные числа равны! Действительно, докажем по индукции, что любые N натуральных чисел равны между собой. При N=1 утверждение верно. Пусть нам известно, что любые N натуральных чисел равны между собой. Возьмем любые (N+1) натуральных чисел. Выкинем любое одно, тогда по предположению индукции все оставшиеся числа равны между собой. Затем вернем это число обратно и выкинем другое, все оставшиеся числа также равны по предположению индукции; в частности, они равны тому, что было выкинуто вначале, что и требовалось.
>> No.71125 Reply
>>71051
Два метода:
1) непосредственное построение объектов, чье существование требуется;
2) составить формальное доказательство (обратись за помощью к учебникам матлогики).
>>71070>>71120
Модус поненс - правило вывода. Их несколько.
>>71068
На его картинке изображен градиент в точке на римановом многообразии.
>> No.71126 Reply
File: -3.png
Png, 16.31 KB, 595×471 - Click the image to expand
edit Find source with google Find source with iqdb
-3.png
Так, на пальцах. Видим последовательность, например. Сначала доказываем "верность" базы. База — это какбэ минимум.
Потом берём варианты этой последовательности для N. Доказываем, что оно верно. И вычитаем/прибавляем один член. Приводим полученное к нужному виду. Если и это верно, то индукционный переход доказан.
Если база верна и индукционный переход тоже, то математическая индукция э-э.. тоже таки верна.
Теперь практика:
Докажем, что все нечётные натуральные числа, начиная с трёх — простые.
Три — нечётное число, так как, если выполнить деление с остатком на два, то тот будет равен одному.
Последующее натуральное нечётное число — 5. А далее 7. Можно заключить, что 3, 5, 7 — составляют арифметическую прогрессию с разностью = 2.
Теперь индукционный переход. Пусть an — нечётное простое число. Это значит, что при выполнении деления с остатком числа an на число < an остаток будет > 0. Обозначим остаток = r.
Пусть n = 4 и an = 9. Выполняем деление с остатком на 2. 2 < 9. 9/2 = (4) + r. r = 1. Верно. Выполняем деление с остатком на 3. 3 < 9. 9/3 = (3) + r. r = 0. 9 — составное число. Значит предположение, что все нечётные натуральные числа, начиная с трёх — простые, неверно.
Фак, индукцию приплести не получилось, вышло доказать только, что все нечётные натуральные числа, начиная с трёх — простые, неверно.

>>71067
Я посмотрел и принцип понял, но не четыре момента; они обозначены на пикрилэйтед.
>> No.71129 Reply
>>71125
> На его картинке изображен градиент в точке на римановом многообразии
ВНЕЗАПНО!
Действительно, не рассмотрел.
Исправляюсь: Ув. тов. >>70564 , то, что на твоей картинке, имеет отношение к градиенту.
>> No.71138 Reply
>>66948
Приветствую кафедру математики!
Аноны, суть такова: я ушел с информатики за неделю до экватора, так как дропнул весь физмат начисто. За три месяца метаний окончательно понял, чего я хочу в учебе - изучать лингвистику. Посмотрев программы, я понял что бида-бида, царицу наук лингвисты изучают изрядно. Ну а раз я упрямец, придется заранее браться за ту область знаний, которую никогда как следует не учил (хотя в школе сдавал на 5).

Посему прошу посоветовать литературы для самоподготовки, на русском или английском, по следующим разделам математики:
  • Математический анализ (Наслышан о Зориче. Правда ли, что это эпик вин?)
  • Высшая алгебра (Куроша я, к стыду своему, не осилил.)
  • Математическая статистика и теория вероятностей (Что-нибудь легкое для восприятия, я коротенький курс этой дисциплины еле на "удв." вытянул.)
Найденные мною учебники по математической логике (Набебин/Кораблин) и дискретной математике (Р. Хаггарти) вроде бы устраивают по уровню подачи материала, не вызывая взрыва мозга, по крайней мере на первый взгляд. Так что приветствуется где-то такой уровень.
>> No.71143 Reply
>>71107
Так он мне их и нарисовал, и спросил, почему так.
>>71091
Да ничему не противоречит, естественно.
>> No.71175 Reply
>>71138
> Наслышан о Зориче. Правда ли, что это эпик вин?
Правда. Но может быть все равно тяжеловат, если всё так запущено; вероятно также понравится Фихтенгольц.
>> No.71183 Reply
>>71175
> может быть все равно тяжеловат
тише, спугнёшь!
Первы том Зорича савсэм простой, да, оче разжёван материалы.
> Фихтенгольц
А вот этого говна не надо, не надо.
>> No.71185 Reply
>>71143
Если человек спрашивает "почему верно A?", его должно удовлетворить доказательство утверждения A, ни более ни менее. Если ему нужно что-то ещё - пусть формулирует другой вопрос.

В данном случае мы доказываем существование, поэтому достаточно (хотя и не необходимо) предъявить объект и доказать, что он обладает требуемыми свойствами.
>> No.71194 Reply
>>71183
> А вот этого говна не надо, не надо.
Почему?
>> No.71199 Reply
>>71183
> разжёван
Лингвисту не надо знать что такое действительное число, ящитаю. Ему даже не надо знать что такое функция. Не надо лингвисту на такое 80 страниц угрохивать 40 в самый раз.
>>71194
Ну там действительно фигня со строгостью изложения же ...
>> No.71202 Reply
>>71199
> Лингвисту не надо
ой, я не обратил внимания, что там лингвист спрашивает, а комментировал сразу тезис о сложности Зорича.
Для лингвиста, наверное, подойдёт
Я. Зельдович, И. Яглом Высшая математика для начинающих физиков и техников
сам её не читал, но многие о ней положительно отзывались

>>71194
Почему?
Потому что он не нужен просто.
>> No.71633 Reply
>>71126
> Я посмотрел и принцип понял, но не четыре момента; они обозначены на пикрилэйтед.
Да, действительно, я перепутал формулу и написал не то.
>> No.72088 Reply
>>70083
> А тензоры, например, это такие наборы чисел, которые при замене базиса изменяются определённым образом, да?
Что не так? Это как-то по другому определяется?
>> No.72276 Reply
>>72088
Он хочет тебя подловить, няша. Понимаешь, математикам на 1-2 курсе неожиданно открывают, что красивый и правильный способ себе мыслить многие объекты - это определять их бескоординатно. Не надо нам никаких R^n, ебашим сразу многообразия. Кто-то сказал теорема Стокса? Нахуй, нахуй, это комплекс де Рама и ниибет. Такой, какой-то математический фундаментализм и бесконечный местный холивар, который наблюдается во всех тредах по математике здесь.
>> No.72279 Reply
>>72276
> Он хочет тебя подловить, няша
А я просто хочу узнать, как будет по-умному.
>> No.72281 Reply
>>72088
Дело в том, что координаты -- не "родной" орган векторного пространства, их нет в определении векторного пространства, это искусственно наращенный фурункул, пятая нога у собаки. При решении детских задачек эта уродливая конструкция может быт полезна, но не более того.
>>72276
Дело не в красоте и даже не в правильности, пересмотр образовательной программы имеет исключительно практический смысл. Обучение полноценного компетентного математика занимает примерно 10 лет (начиная со старших классов и заканчивая первыми годами постдока). Тратя время на "аналитическую геометрию" и прочее говно мамонта человек отнимает сам у себя время, которое мог бы потом (если бы сейчас занимался делом) использовать для изучения актуальных вещей.
>> No.72282 Reply
Позанимайтесь со мной математикой. Я хочу быть умным, как и вы. Хнык-хнык.
>> No.72284 Reply
>>71175
>>71202
Спасибо, няши!
Буду откапывать Зорича и посмотрю Зельдовича/Яглома.
>> No.72285 Reply
Кстати да, аноны. Посоветуйте что-нибудь по математической логике, желательно версию для альтернативно одарённых. Только учтите, что я и таблицу умножения нетвёрдо знаю, ибо тян .
>> No.72288 Reply
>>72281
> > А тензоры, например, это такие наборы чисел, которые при замене базиса изменяются определённым образом, да?
> Что не так? Это как-то по другому определяется?
Оно выглядит примерно так: верхние индексы изменяются по правилам касательного (вектора), а нижнее по правилам кокасательного расслоения. И это хорошо, например, в физике, потому что там везде метрический тензор. С ним сразу понятно как работать. Производная Ли сразу же считается. Ковариантная связность, хоть и не тензор, но выглядит понятно. Короче, можно сразу СЧИТАТЬ. И если что-то забыл, то просто вспомнить. Это наглядный способ. Более того, через некоторое время становится понятно, почему же он ущербный в более сложных конструкциях. Поэтому педагогически кондовая совковая программа имеет смысл. Сразу ебашить хардкор может не хватить сил. Вербятке похожие аргументы тысячу раз говорили (там была другая тема: зачем ты полгода объясняешь меру Лебега, если в практическом плане это нахуй не нужно, все интегралы считают по риману).

Я не сторонник мехматовских дедов-шизиков, но университется программа не так уж плоха, как ты говоришь, прозелит.
>> No.72290 Reply
>>72288
> верхние индексы изменяются по правилам касательного (вектора)
> а нижнее по правилам кокасательного расслоения
> кокасательного расслоения
Не понимат.
>> No.72293 Reply
>>72288
Адресовал я, конечно, тебе >>72281
>> No.72296 Reply
>>72288
> Короче, можно сразу СЧИТАТЬ.
> практическом плане это нахуй не нужно, все интегралы считают по риману
Дело просто в том, что занятия математикой не требуют ничего считать.
>> No.72298 Reply
я имею ввиду, конечно, вычисление конкретных значений, взятие конкретных интеграл и т.д.
>>72296
>> No.72323 Reply
File: h-index.jpg
Jpg, 62.84 KB, 610×691 - Click the image to expand
edit Find source with google Find source with iqdb
h-index.jpg
>>72296
Это какой-такой областью ты занимаешься, что от тебя не требуется обладать execution power? Хотя блин.
> Дело просто в том, что занятия математикой не требуют ничего считать.
"Нематематики считают, что математики считают."

Энивэй, по мимо пиздобольства всегда требуется что-то посчитать. И конкретный результат всегда радует и душу, и рецензента. Так что лучше учиться СЧИТАТЬ сразу же, а то вырастешь философом, а не математиком.
>> No.72337 Reply
>>72323
lol'd
Открой какой-нибудь раздельчик в Архиве, AG или DG, например, да посмотри там парочку статей
>> No.72581 Reply
Посоветуйте годную книжку по матану.
У меня лежала советская "Руководство к решению задач по матану", но я её оставил в другом городе, а заниматься нужно.
>> No.72594 Reply
>>72581
Теория вот: >>67496
Из задачников знаю только Кудрявцева, но там, кажется, всё хорошо разъяснено, с примерами.
>> No.72595 Reply
>>72281
> Дело в том, что координаты -- не "родной" орган векторного пространства
Если уж сводить обсуждение к органам, то скалярное произведение - гораздо менее "родной" орган векторного пространства, чем координаты. Оно вообще не его орган.
>> No.72597 Reply
>>72595
Сделал мой день.
>> No.72928 Reply
Прорешал главу в задачнике, стал пересматривать через месяц и оказалось, что я почти ничего не запомнил. Иногда бывает, что уже через день смутно помню решение задачи. И, если не смог решить в прошлый раз, то не могу решить и в этот.
Аноны, это нормально? А то у меня из-за этого возникает сильное ощущение зря потраченного времени. Может я как-то не так занимаюсь? А то все решаю да решаю, а навыков почти не прибавляется.
Ну и вообще, как ВЫ учите математику?
>> No.72935 Reply
>>72928
> Аноны, это нормально?
Нет.
А что ты решаешь?
Если какую-нибудь вычислительную муть, то это как раз наоборот -- здоровая реакция организма.
>> No.72938 Reply
>>72935
> А что ты решаешь?
Да школьные задачи - тригонометрия, алгебра.
Меня больше беспокоит не то, что я не помню сами решение, а то, что у меня не остаются в голове приемы решения. То есть арсенал методов у меня довольно скуден.
>> No.72941 Reply
> Если какую-нибудь вычислительную муть, то это как раз наоборот -- здоровая реакция организма.
Как раз наоборот - задачи весьма оригинальны.
И еще хотел спросить: как по-вашему, 3-4 задачи средней-высокой сложности в час - это нормальная скорость?
>> No.72946 Reply
>>72941
> 3-4 задачи средней-высокой сложности в час - это нормальная скорость?
Смотря что считать сложными задачами - если школьные задачи с не совсем тривиальным решением - то скорость нормальная.
>> No.72955 Reply
Школьные задачи, и 3-4 примера в час... Мало что запоминается, навык не прогрессирует?
Я бы не сказал, что это нормально или хорошо. Но и некритично. Навык, может быть, рано или поздно отточится.
>> No.72956 Reply
>>72928
> Ну и вообще, как ВЫ учите математику?
Я учу днём, учу ночью - где-то день-два. А на следующее утро иду на экзамен.
Как правило, всё запоминается - особенно если дальше учить, там ж одно на другом строится.
>> No.72957 Reply
>>72938
> алгебра
А что сейчас входит в школьную программу алгебры?
Подозреваю какие-нибудь "33 оригинальных способа решить кубическое уравнение"
В общем, это я и назвал вычислительной мутью (тригонометрия ею является по определению)
====
> N задач в час
Какое это имеет значение? Ты же не на тренировке.
>> No.73041 Reply
>>72935
То, что ты называешь "вычислительная муть" - это единственная практически полезная часть математики. Ты можешь дать ответ вот на такую задачу:
"Воздушный шар летит в одном направлении со скоростью 20 км/час в течение 1-го часа и 45-ти минут. Затем направление движения меняется на заданный угол (60°), и воздушный шар летит ещё 1 час и 45 минут с той же скоростью. Найти расстояние от точки старта до точки приземления"?
>> No.73054 Reply
>>73041
ага-ага
расскажи мне о практической пользе, диспетчер
>> No.73091 Reply
>>73041
> "вычислительная муть" - это единственная практически полезная часть математики.
Толсто
>> No.73094 Reply
>>72941
Анон, кинь одну «оригинальную».
>> No.73095 Reply
>>73041
> "Воздушный шар летит в одном направлении со скоростью 20 км/час в течение 1-го часа и 45-ти минут. Затем направление движения меняется на заданный угол (60°), и воздушный шар летит ещё 1 час и 45 минут с той же скоростью. Найти расстояние от точки старта до точки приземления"
Шутник штоле?
Получается равносторонний треугольник, соответственно расстояние от точки старта до точки приземления равно 35 километров. >:3
>> No.73096 Reply
>>73095
Не, там треугольник с углами 120, 30, 30, и получается 35 корней из трёх.
В любом случае задачка для восьмого класса где-то. Странно, что >>73041 пытается нас ей удивить.
>> No.73098 Reply
File: riangle.png
Png, 10.09 KB, 400×400 - Click the image to expand
edit Find source with google Find source with iqdb
riangle.png
>>73096
> Не, там треугольник с углами 120, 30, 30, и получается 35 корней из трёх.
Не понимат. С чего ты взял?
Рассматриваем двумерную плоскость.
Сначала шар пролетает одно расстояние: 35 км. От A до B.
Потом меняет направление на 60 градусов и точно также пролетает ещё 35 км. От B до C.
Соединяем A и C. Получается равносторонний треугольник.
Пролетел шар 70 км, а расстояние между A и C = 35 км.
>> No.73099 Reply
>>73098
На твоей картинке направление меняется на 120 градусов - возьми хоты бы скалярное произведение между векторами начальной и конечной скоростей.
>> No.73104 Reply
File: Безымянный.jpg
Jpg, 4.34 KB, 244×39
edit Find source with google Find source with iqdb
Безымянный.jpg
File: p0042-sel.bmp
Bmp, 22.45 KB, 2839×671
edit Find source with google Find source with iqdb
p0042-sel.bmp
File: p0037-sel.bmp
Bmp, 3.11 KB, 1213×176
edit Find source with google Find source with iqdb
p0037-sel.bmp

>>73094
На. Первые 2 из конца задачника.
>> No.73124 Reply
>>73104
это всё бесполезная хуета, называется "вступительная математика".
Оно может пригодиться при поступлении в техновуз, но к изучению науки такие задачи отношения ни малейшего не имеют.
>> No.73132 Reply
>>73124
> но к изучению науки такие задачи отношения ни малейшего не имеют
Ок, что такое "изучение науки"? Пока что я склонен считать, что это термин, которым прикрываются математические импотенты, которые тройного интеграла взять не могут и придумывают всякую псевдоматематичную муть.
>> No.73134 Reply
>>73132
вырастешь большим троллём -- станешь леса королём!
>> No.73135 Reply
>>73134
До-до. Вместо ответа - оскорбления.

>>73104
Няша, не слушай их, работай над задачником.
>> No.73150 Reply
>>73135
lol'd
какое слово тебе разъяснить: "изучение" или "наука"?
>> No.73154 Reply
>>73104
Первую задачку в уме можно решить.
>> No.73157 Reply
File: 1344760629705.jpg
Jpg, 14.31 KB, 188×267 - Click the image to expand
edit Find source with google Find source with iqdb
1344760629705.jpg
Привет, матач.
Подскажи, пожалуйста, у меня дилемма.
В общем, подумываю поступить на второе высшее, а именно - прикладная математика. Форма обучения в ВУЗе, куда хочу отнести документы - дистанционная, в течение 2-х лет.
За плечами имею диплом бакалавра и магистра в инженерно-технической области, закончил учебу в 2011, т.е. сейчас мне 24 года.
Проблема в том, что вся математика, которую я на "5" знал в школе и весь элементарный матан первых двух курсов института я уже забыл (общие понятия остались, конечно, но решить какой-нибудь предел я сейчас не смогу).
И я теперь перед выбором - либо мне идти поступать на эту специальность и надеяться, что за 2 года дистанционного образования я вспомню и узнаю хоть что-то, и смогу совмещать учебу со своей работой (IT); или же подождать годик, почитать всяких Письменных, Выгодских и вспомнить всё, что когда-то знал, и уже с такой базой поступать.
Еще, конечно, вопрос - будет ли толк от СНГшного заочного мат.образования, и смогу ли я с этим дипломом претендовать на место, например, научного сотрудника в каком-нибудь НИИ в Европке или Америках?
Спасибо.
>> No.73161 Reply
>>73150
> что такое "изучение науки"?
>> No.73164 Reply
>>73154
Ну реши.
>> No.73166 Reply
>>73161
в этом словосочетании нет никакого скрытого смысла, его надо понимать буквально
"пить воду", "сидеть на стуле", "изучать науку"
Я поэтому и спросил значение какого из слов тебе непонятно.
>> No.73169 Reply
>>73157
> будет ли толк от СНГшного заочного мат.образования, и смогу ли я с этим дипломом претендовать на место, например, научного сотрудника в каком-нибудь НИИ в Европке или Америках?
Это шутка? Очевидно, что нет.
Наука в мире и "наука" в СНГ (и в Рашеньке в том числе), это два совершенно различных рода деятельности, которые вообще никак не пересекаются (ещё сохранились оазисы культуры, которые готовят аспирантов для западных вузов, но их можно пересчитать по пальцам одной руки).
>> No.73204 Reply
>>73164
Ясно, что если x=1-5x^2, то этот x удовлетворяет и исходному уравнению (ибо подставим верное равенство x=1-5x^2 в правую часть верного равенства x=1-5x^2). Отсюда находим два корня исходного уравнения четвертой степени, потом делим.

По-взрослому это звучит так: если точка x является неподвижной точкой отображения x|->f(x), то она является и неподвижной точкой отображения x|->f(f(x)).

мимопроходил

Стараюсь писать без юникодностей, а то некоторые жалуются. Или лучше с юникодностями?
>> No.73205 Reply
>>73166
Ок. Что нужно сделать, чтобы изучить математику?
>> No.73206 Reply
>>73205
Смотря с какой целью: использовать математику как инструмент в другой науке или заниматься непосредственно математикой.
Я готов дать ответ только на второй вопрос.
>> No.73207 Reply
>>73104
В 120. просто приводишь к общему знаменателю и схлопываешь в числителе очевидный синус суммы.
>> No.73215 Reply
>>73169
Хорошо, как тогда быть человеку (мне, то бишь), который хочет заниматься современной математикой? Куда поступать (я живу в КЗ, к сожалению), есть ли программы дистанционного образования в зарубежных ВУЗах, или том же НМУ?
>> No.73219 Reply
>>73206
Ок, давай.
>> No.73224 Reply
>>73104
Так и не вижу красивого и правильного решения 257ой. Рабочий вариант - записать теорему Виета для двух уравнений, получить систему на три корня двух уравнений, она легко решается, подставить полученные рациональные выражения в третье уравнение и схлопнуть.
>> No.73228 Reply
>>73219
>>73215
Отвечу одним сообщением на оба вопроса.
> заниматься современной математикой?
Возможно, под "современной математикой" мы имеет ввиду разные вещи.
Я пользуюсь функциональным определением: современная математика это то, что публикуется в основных (алгебра/геометрия/топология) разделах на сайте
http://arxiv.org/archive/math
А математиком называется тот человек, который без подготовки в состоянии понять большую часть статей из указанных разделов.

Далее, если ты хочешь занимать именно математикой (а не CS, инженерными науками или физикой), то выбрать место учёбы очень просто, ибо их меньше, чем нихуя.
> есть ли программы дистанционного образования
многие западные университеты выкладывают конспекты и видеозаписи лекций (и их непременно нужно изучать!), но, чтобы куда-то потом устроиться, нужна и бюрократическая бумажка, т.е. диплом.
> Куда поступать
В твоём случае (повторюсь, если речь идёт о математике!) только НМУ.
Чтобы получить диплом нужно сдать 24 курса и написать диплом. Курс считается сданным, если ты написал экзамен и сдал нужное количество задач. В твоём случае можно связаться с преподавателями курсов, которые ты собираешься сдавать, и договориться о сдаче задач во время сессии (по умолчанию считается, что студент сдаёт задачи каждую неделю). В общем система немного запутанная, но она предоставляет максимальную свободу желающим учиться.
У тебя действительно серьёзные планы? А лет сколько?
И сразу демотиватор: учиться дистанционно было много желающих, но все сдулись
>> No.73232 Reply
>>73228
Планы-то серьезные, но, похоже, условий у меня здесь нет инбифо ленивая скотина. 24 года только.
Так в НМУ есть дистанционка, или ты про сам подход в общем?
Кроме того, как я понял, мне нужно самостоятельно повторить школьную программу и основную математическую программу вуза за 2-3 года, чтобы не "лажануться" в НМУ?
>> No.73273 Reply
File: 2022964.jpg
Jpg, 39.83 KB, 300×400 - Click the image to expand
edit Find source with google Find source with iqdb
2022964.jpg
>>73232
> Так в НМУ есть дистанционка?
Нет. Если хочешь учиться то на сессии (2 раза в год) тебе придётся в любом случае приезжать в Москву и писать экзамены (хотя некоторые экзамены выдаются на дом). Можно попробовать договориться с преподами, чтобы во время этих твоих приездов они у тебя оптом приняли задачки за весь курс (сдача задач -- необходимое условие, чтобы курс был зачтён), листочки с задачами еженедельно выкладываются на сайте по мере чтения курсов.
Ну и ещё чтобы губу не раскатывал к концу первого года где-то 90% когда-то желающих учиться бросают НМУ, хотя у них есть возможность почти неограниченно общаться с преподами и другими студентами, а уж сидя дома мотивация пропадёт ещё быстрее но мне хотелось бы верить, что ты действительно серьёзно натсроен.
> мне нужно самостоятельно повторить школьную программу и основную математическую программу вуза
Нет, лучше забыть эту гадость как страшный сон, для изучения математики от перечисленного дерьма больше вреда, чем пользы.
Вообще в НМУ вся теория излагается "с нуля" и предварительных знаний не требуется (кроме самых-самых тривиальностей), но требуется умение решать задачки, т.е. чтобы от слов "докажите, что ..." у тебя не начинала кружиться голова.

В качестве действительно хорошей подготовки могу порекомендовать пикрелейтед.
"Теорема Абеля в задачах и решениях", Алексеев
Эта книжка содержит лекции/задачи для матшкольников, некогда прочитанных одним из сильнейших математиков ХХ века.
прорешав её ты будешь знать математику лучше трети выпукников мехмата, лол
>> No.73325 Reply
>>73273
> для изучения математики от перечисленного дерьма больше вреда, чем пользы.
Почему? В том же НМУ есть курсы матана, теорвера и диффуров.
> прорешав её ты будешь знать математику лучше трети выпукников мехмата, лол
Но на мехмате тоже есть курс алгебры.
>> No.73328 Reply
>>73325
> Почему? В том же НМУ есть курсы матана, теорвера и диффуров.
Это прекрасные науки, дело не в них, а в том как они преподаются.
ну и, к слову, классический курс диффуров и теорвер, мягко говоря, не пользуются популярностью
>> No.73333 Reply
>>73328
Но в списке литературе к курсу матана написаны обычные Зорич и Фихтенгольц. В чем различие?
>> No.73337 Reply
>>73333
> В чем различие?
Различие в том, что в технических вузах (и в большинстве случаев на мехмате) "возьмите 50 производных", "возьмите 50 интегралов" и определение градиента через набор частных производных, а с другой стороны качественные задачи (это если говорить конкретно об анализе).
> Фихтенгольц
ВНЕЗАПНО! Действительно, да ещё на первом месте в списке. Да, бывают и такие казусы. Того, кто будет читать анализ я не знаю, хуёвые преподы и в НМУ не редкость (хороших, к счастью больше), благо всегда есть возможность на него просто не ходить.
>> No.73340 Reply
>>73337
Что ты подразумеваешь под качественными задачами?
>> No.73341 Reply
P.S.
А вот алгебру и геометрию в этом году очень приятные люди читают, и рекомендуемая литература годная.
Про Вьюгина не могу сказать ничего плохого, потому что не знаю его, но Фихтенгольц настораживает, лол
>> No.73346 Reply
>>73340
> подразумеваешь под качественными задачами?
когда ты смотришь в методичку, чтобы найти алгоритм решения -- это пустая трата времени
а когда ты придумываешь что-то качественно новое (например, ищешь доказательство какого-то факта), то такая задача называется, соответственно, качественной
>> No.73355 Reply
Ну, это, конечно, утрированное объяснение.
Задрачивание "оригинальных" задач на тригономтерические формулы, конечно тоже пустое занятие.
Качественная задача имеет своей целью углубить знание предмета, а её противоположность, соответственно, -- вбить в голову алгоритм/формулу.
>> No.73368 Reply
>>73355
Как-то встретил сборник для школоты-57, там почти все такие. Пичалька, что учился в обычной школе.
>> No.73375 Reply
Первое предположение с потолка:
> Ясно, что если x=1-5x^2, то этот x удовлетворяет и исходному уравнению (ибо подставим верное равенство x=1-5x^2 в правую часть верного равенства x=1-5x^2). Отсюда находим два корня исходного уравнения четвертой степени, потом делим.
А второе:
> По-взрослому это звучит так: если точка x является неподвижной точкой отображения x|->f(x), то она является и неподвижной точкой отображения x|->f(f(x)).
Более взрослый матан?

Йоптыть, второе сразу вкурил и понял. Даже понравилось. А над первым думал минут десять, пока не дошло о чём речь.
Блджад, мне начинает нравиться «взрослая» математика.
>> No.73377 Reply
>>73337
> и определение градиента через набор частных производных
Градиент определяется по-разному, но выводят его всегда через производную по направлению. Покажи место, где последнее обставлено иначе.
>> No.73387 Reply
>>73375
И то и другое по-русски надо записать так: если x=f(x), то x=f(f(x)).
Никакого более глубокого смысла тут нет.
Контрольный вопрос: верно ли обратное?
>> No.73392 Reply
>>73387
Только если отображение биективно, не?
>> No.73400 Reply
>>73368
К сожалению, дрочить задачи на вступительную тригонометрию тоже приходилось в больших объемах, чтобы поступить (большинство поступило по олимпиаде и не пригодилось, такая вот тщета).
Мимо 57-ит
>> No.73425 Reply
>>73392
А вот и нет. Для f(x)=x^2 это тоже верно.
>> No.73426 Reply
>>73375
> Ясно, что если x=1-5x^2, то этот x удовлетворяет и исходному уравнению (ибо подставим верное равенство x=1-5x^2 в правую часть верного равенства x=1-5x^2). Отсюда находим два корня исходного уравнения четвертой степени, потом делим.
> потом делим.
А? Откуда взялось деление? Что на что делим?
>> No.73428 Reply
>>73426
Ну вот у тебя уравнение четвертой степени, ты знаешь два его корня. Как найти два других? Проще всего поделить многочлен столбиком.
>> No.73431 Reply
>>73400
А где сейчас учишься?
>> No.73437 Reply
File: 1344937103846.png
Png, 1.02 KB, 300×20 - Click the image to expand
edit Find source with google Find source with iqdb
1344937103846.png
>>73273
Судя по всему, ты хорошо во всей этой кухне разбираешься, потому задам главный вопрос, которым задаются все, кто хочет постигать науку математики с нуля.
Какой геометрический, и, что более важно, физический смысл есть у всех записей в матанализе и смежных дисциплинах (топологии, все эти тензоры, частные производные, комплексные числа, какие-то "гладкие поверхности", Лагранжи, группы Ли, и т.д. и т.п.)?
Человек вряд ли будет изучать то, в чем не видит смысла - это его инстинктивно оттолкнет. Так какой у всего этого смысл? Я понимаю, что матлогика качает мозг. Но остальное? Какое прикладное значение у всего этого? Пардон за эмоциональность, просто это очень важно понять
КЗ-кун
Капча доставила.
>> No.73441 Reply
>>73437
Твой вопрос звучит как "каков физический смысл умножения вещественных чисел"? Очевидно, он может быть каким угодно. Давай лучше попросим у сотрудников кафедры (должен же хоть один из них быть прикладником!) показать пример физических и экономических задач, которые решаются с помощью:
  • комплексных чисел
  • двойных интегралов
  • тройных интегралов
  • групп Ли
  • тензоров
  • дифференциальных уравнений
>> No.73445 Reply
>>73441
> Твой вопрос звучит как "каков физический смысл умножения вещественных чисел"?
Скорее, "каков физический смысл манипуляций с математическими абстракциями?", если я правильно выразился.
Но ты согласен со мной в том, что без видимого смысла занятия математикой не будут продуктивными?
>> No.73450 Reply
>>73445
> каков физический смысл манипуляций с математическими абстракциями?
Вот, на примере умножения. Пусть у нас есть пять авосек, в каждой авоське по десять яблок. Тогда 510 = 50 будет общим количеством яблок. Или вот: пусть у нас есть прямоугольная комната размерами шесть на восемь метров, тогда 68=48 метров - это её площадь. Как видишь, числу, получающемуся в результате умножения, соответствуют различные физические объекты - и яблоки, и квадратные метры. Выходит, один-единственный физический смысл умножения выделить не получится.
Ты, как я понимаю, просишь хоть какой-нибудь смысл? Для интегралов можешь прочитать статью "механические приложения интегралов" в вики, для тензоров - статью "тензор проводимости" в ней же. Ещё можно подождать, вдруг кто-нибудь обстоятельно ответит на просьбу из >>73441 .
>> No.73453 Reply
>>73441
Просто зайди на Википедию, в статьях по этим терминам и приведены примеры использования.
>> No.73462 Reply
>>73453
У групп Ли не приведены, например.
>> No.73469 Reply
>>73437
Твой вопрос труден очень.
====
Понятно, почему люди хотят заниматься физикой или биологией. У начинающего заранее есть представление о своей науке, он что-то слышал про чёрные дыры, генную инженерию, тёмную материю и хочет в этом разбираться. Естественно, представление начинающего о науке не имеет ничего общего с самой наукой, но оно даёт мотивацию к занятиям.
И вот ты спрашиваешь, какова мотивация человека, приступающего к изучению математики. Вопрос справедливый, потому что начинающий (в отличие от представителей других наук) принципиально не может понять, чем занимаются практикующие математики (способность это понять, как я писал выше, собственно, и является свойством, дающим право называться математиком).

Ответа на этот вопрос я не знаю.
Во-первых важным, (а иногда решающим) фактором является пример родителей и/или учителей.
Во-вторых школьник, выбирающий профессию математика, интуитивно чувствует, что приобщается к чему-то важному, чистому, объёмному, весомому.
В-третьих (это самое главное), людям просто нравится сам процесс понимания математики, люди находят удовольствие в том, что в голове выстраивается структура.
====
Потом ты спрашиваешь про практическую пользу. Этот вопрос намного легче.
Дело в том, что "практическая польза" это просто игра слов, которую представители разных наук по очереди используют друг против друга, чтобы отбить финансирование в свою пользу (когда финансирования хватает всем, про практическую пользу никто не вспоминает, лол)
Имеет ли практическую пользу теория чисел? Математики начала ХХ века горда заявляли, что она совершенно бесполезна. А сейчас на теории чисел построена вся криптография и она находит неожиданные применения в самой передовой теоретической физике.
Поиск "практической пользы" это ложная и недальновидная мотивация, нужно просто разобраться в себе, что больше по душе: делать что-то руками (и быть программистом, инженером, экспериментатором) или быть теоретиком.
====
Тогда можно задать вопрос, если физик-теоретик или CS-теоретик хотя бы претендуют на то, что изучают реальные вещи, то что изучает математик-теоретик?
Я (и многие-многие другие) придерживаюсь того мнения, что математик изучает, соответственно, математическую реальность. Математические объекты и структуры не рукотворны, а существуют независимо от того, известны они людям или ещё нет. А учёные открывает эти структуры и взаимосвязи между ними подобно тому, как археолог выкапывает из под земли древности или путешественник составляет карту местности. Математическая реальность ничем не лучше и не хуже физической (и других) и заслуживает не меньшего внимания и изучения.
такие дела
>> No.73475 Reply
И сразу добавлю, что ни в коем случае не нужно загонять себя в рамки типа "я математик, а не инженер" или "я инженер и не буду изучать геометрию".
Например, почти все отцы-основатели CS были математиками с классическим университетским образованием и вели свои разработки параллельно с работами по геометрии, топологи и т.д.
>> No.73477 Reply
>>73469
Твоя точка зрения весьма интересна; но хотелось бы всё же увидеть выходы математики в реальный мир. Математика в твоём описании похожа на схоластику, тогда как она, судя по всему, весьма жизненна. Покажи приложения математики, пожалуйста.
>> No.73478 Reply
>>73477
> реальный мир
"Реальный мир" это фикция. Когда я представляю двумерную сферу она для меня не менее реальна, чем стопка книжек на столе.
О "реальном мире" может говорить только религия или философия. Физика. к примеру, изучает свои модели, совокупности которым составляют физическую реальность, но это не имеет ничего общего с "реальным миром".
>> No.73479 Reply
>>73478
Ок. Как используются группы Ли в физике?
>> No.73485 Reply
File: huge.61.305562.JPG
Jpg, 23.54 KB, 450×374 - Click the image to expand
edit Find source with google Find source with iqdb
huge.61.305562.JPG
>>73479
Доставлено:
enwiki://Particle_physics_and_representation_theory
Алсо физики налегают на группы ЛИ впятеро больше студентов-математиков (кроме тех, кто особенно угорел по теории представлений)
>> No.73494 Reply
Добрый день математики. Хочу ради собственного развития изучить математику хотя бы школьного уровня. Будет ли сложно? Нужно ли это вообще? С чего начать?
>> No.73495 Reply
>>73494
Платиновые вопросы доброчана
Где-то в предыдущих тредах уже спрашивали и отвечали.
>> No.73496 Reply
>>73469
> Вопрос справедливый, потому что начинающий (в отличие от представителей других наук) принципиально не может понять, чем занимаются практикующие математики
Хорошо. Через что приходит принципиальное понимание математики? Полагаю, не через монотонное чтение фихтенгольцев?
> люди находят удовольствие в том, что в голове выстраивается структура
Это что-то вроде искусства ради искусства? Не могу этого понять, я, видимо, прикладник по натуре.
> Математические объекты и структуры не рукотворны, а существуют независимо от того, известны они людям или ещё нет.
Что математику-теоретику дает знание этих структур? Вот про теорию чисел отличный пример ты привел. А какие-нибудь "гладкие многообразия"? Может ли математик, владеющий этой (или любой другой) теорией прийти к знакомому физику и сказать "о, прикинь, оказывается то, что ты не можешь посчитать в своей модели считается вот так-то и так-то, и смотри, что у тебя все сходится!"? Или "о, перестроив это уравнение так-то и так-то, мы сможем создать модель %рандомноесобытиеname%"?
>> No.73498 Reply
>>73495
Ну так FAQ надо составить.
Хотя с другой стороны не настолько и популярный тред. И он смотрится по меньшей мере странно среди тредов про православие, буддизм, атеизм, ОСы, воображаемых друзей и "мы безвольные мудаки, давайте поддержим друг друга".
Извините, накипело.
>> No.73499 Reply
>>73494
> Будет ли сложно?
Если делать это системно и, желательно, с кем-нибудь на пару - нет. А вообще, основы матана может понять каждый.
> Нужно ли это вообще?
Есть сильно прикладные области математики (теория вероятностей, теория игр, матанализ и некоторые другие), а есть те, которые используются более редко. "Математика ум в порядок приводит".
> С чего начать?
> с школьного уровня
>> No.73501 Reply
>>73477
Ну есть некая знаменитая копипаста на эту тему, называется Е. Вигнер "Непостижимая эффективность математики в естественных науках (физика наших дней)". Нагуглите, хорошо пишет няша.
>> No.73502 Reply
File: t57.jpg
Jpg, 44.51 KB, 600×191 - Click the image to expand
edit Find source with google Find source with iqdb
t57.jpg
>>73498
Не обижай мой тред. Он хороший. Для верцунов и сновидцев есть кнопка "скрыть". :3
Пойду ещё физику бампну.
ОП
>> No.73505 Reply
>>73469
> находит неожиданные применения в самой передовой теоретической физике.
С этого места поподробнее.
>> No.73506 Reply
>>73502
А ты которого треда автор?
>> No.73510 Reply
>>73496
> Через что приходит принципиальное понимание математики?
Чтение книжек, усердное решение качественных задач, обсуждение с другими людьми (это очень важно!). Посещение конференций и разбор статей из Архива, когда появятся силы.
> > в голове выстраивается структура
Я имел ввиду само субъективное чувство удовольствия. Мне, например приятно понимать структуру галактики или структуру человеческого тела. Но это понимание структуры не полное, я упускаю множество деталей, потому что мне известна только модель. А в математике имеет место "кристально чистое", "эталонное" понимание, это чувство я бы назвал интеллектуальной педантичностью.
Кроме того сам факт решения задачки (т.е. внезапный инсайт, открытие) субъективно тоже очень приятен и увлекает.
> Что математику-теоретику дает знание этих структур?
Наука имеет своей целью изучение того, чему можно дать определение, т.е. построить модель.
Астроном изучает галактики, биолог изучает колонии бактерий, математик изучает трёхмерные открытые многообразия, например.
> гладкие многообразия
Чтобы не обсуждать дальше тривиальностей, скажу просто, что вся современная математика давно эксплуатируется физиками и, более того, разрабатывается совместно с физиками.
А "гладкие многообразия" это почти второе имя классической механики.
>> No.73511 Reply
File: fantasy115.jpg
Jpg, 145.85 KB, 1024×768 - Click the image to expand
edit Find source with google Find source with iqdb
fantasy115.jpg
>>73499
Ок, тогда буду осваивать, а непонятное спрашивать. А вы не будете надо мной смеяться?
>> No.73513 Reply
>>73506
Этого и предыдущего.
У кого-нибудь есть книжка Ландау "Основы анализа"? Во всех сканах, которые я нагуглил, есть баг в предисловии и отсутствует с десяток листов в главе про вещественные числа.
>> No.73514 Reply
>>73510
> обсуждение с другими людьми (это очень важно!)
Вот черт, а без это никак?
>> No.73516 Reply
Котаны, а кому-нибудь интересна финансовая математика? Хотя по построению больше на физику смахивает. Стохастически анализ, мартингалы и прочие более простые ништяки. И сам по себе предмет интересный, и в хозяйстве пригодится.

Я сам как-то увлекся, прочитал несколько толковых книжек по ценообразованию деривативов. Но потом из-за учебы забил. Сейчас думаю, все по-новой изучать, на этот раз уже для сдачи professional risk manager exams.
>> No.73518 Reply
>>73496
> Это что-то вроде искусства ради искусства?
Именно. Это особая математическая магия.
>> No.73521 Reply
>>73514
Даже суровые математики вынуждены общаться с другими людьми.
Поверь, в других науках всё гораздо хуже будет для тебя с этим.
>> No.73525 Reply
>>73513
Так я ж не математический тред ругаю, а как раз засилье православнутых со всякими диванными атеистами.
>> No.73531 Reply
File: 11725872.jpg
Jpg, 14.42 KB, 250×256 - Click the image to expand
edit Find source with google Find source with iqdb
11725872.jpg
>>73511
Будем, но необидно. Сами такими были.
>> No.73541 Reply
File: images.jpg
Jpg, 6.83 KB, 259×194 - Click the image to expand
edit Find source with google Find source with iqdb
images.jpg
Матанцы, может, тогда вы поможете составить список книг, которые нужны для изучения этой прекрасной науки с азов, до каких-нибудь вершин?
Ну, например, для начала учебник, в котором вся школьная программа и часть институтской (матрицы, определители, двойные-тройные интегралы и пр.), потом (и это важно!) нужно прочесть что-то, чтобы безболезненно перейти от понятных простому смертному диффуров к мат.символике и новым дисциплинам, которые при первом взгляде вызывают шок.
Ну а уже потом всякие "Арнольды и Фихтенгольцы", наверное. Помогите, пожалуйста.
>> No.73542 Reply
>>73541
> обы безболезненно перейти от понятных простому смертному диффуров к мат.символике и новым дисциплинам
Если ты про кванторы, множества и следования, то с этого лучше начать. Если нет, то я не понимаю, о какой ты символике.
>> No.73546 Reply
>>73541
Самые-самые азы:
Мордкович: "Алгебра и начала анализа", 10-11 классы
Зорич: "Математический анализ" в двух томах
Головина: "Линейная алгебра и некоторые её приложения"
Курош: "Курс высшей алгебры"
В.Б. Алексеев: "Теорема Абеля в задачах"
Арнольд: "Геометрия комплексных чисел, кватернионов и спинов"
Ефимов, Розендорн: "Линейная алгебра и многомерная геометрия"
Гурса: "Курс математического анализа"
>> No.73550 Reply
>>73505
Есть предположение, что при самых маленьких масштабах расстояние принимает значение не в действительных числах, а в p-адических, у них совершенно другая геомтерия, топология и т.д. Подробности надо искать в работах Виттена, а сам я далёк от физики, увы.
>>73514
Можно же общаться в интеренте.
Вот чятик для новичков: http://math.stackexchange.com/
> С чего начать?
> с азов, до каких-нибудь вершин?
> Нужно ли это вообще?
Я могу попробовать составить FAQ, но какие вопросы нужно осветить?
>> No.73554 Reply
File: int.png
Png, 0.76 KB, 117×44 - Click the image to expand
edit Find source with google Find source with iqdb
int.png
Исследовать на сходимость.
>> No.73558 Reply
Надеюсь, при создании следующего треда будет указано, что это не место для обсуждения домашних задач.
>> No.73559 Reply
>>73554
Тон-то какой, аж приказной. "Эй, холопы! Исследовать на сходимость!"
>> No.73567 Reply
>>73558
Пока что треды состоят из срачей насчет терминологии, пользы математики и "что почитать, чтобы стать математиком", так что не вижу ничего плохого в разбавлении треда задачами (желательно интересными). Не утверждаю, что предложенная мной задача является интересной, ибо решить пока не получилось (видимо, туповат).
>>73559
Просто скопировал условие.
>> No.73569 Reply
>>73558
Почему же? Место, если задачи интересные.

>>73554
Иди на вольфрамальфу.
>> No.73574 Reply
>>73569
> Иди на вольфрамальфу.
Не помогло.
>> No.73575 Reply
>>73574
Пруф:
http://www.wolframalpha.com/input/?i=\int_{0}^{\infty}\frac{\sin+x}{x%2B\ln+x}dx
>> No.73598 Reply
>>73554
Это ж банальный признак Дирихле.
А про левый конец я вообще молчу.
>> No.73600 Reply
>>73542
> Если ты про кванторы, множества и следования, то с этого лучше начать. Если нет, то я не понимаю, о какой ты символике.
Ну, наверное про это. И вообще, что-то такое, где расписано, что за буквенными обозначениями скрывается. Ну, понятно, что R - рациональные числа. А таких букв там сотни. Вот такой еще знак ":=".
>>73546
А в каком порядке?
>> No.73602 Reply
>>73600
Это означает "по определению".
В википедии есть вроде таблица математических символов, тебе должно хватить, чтобы логическую символику понимать. Если ты, конечно, про нее, а то я так и не понял, что конкретно тебе недоступно.
>> No.73605 Reply
>>73602
> Это означает "по определению".
Извини, если глупый вопрос, но чем это отличается от простого знака "=" ?
>> No.73606 Reply
>>73501
Вигнер был неправ. В корне.
Кто желает пообсуждать?
>> No.73610 Reply
>>73598
А какие функции ты рассматриваешь?
>> No.73611 Reply
>>73600
> Ну, понятно, что R - рациональные числа.
Но ведь они действительные. Рациональные, кажется, Q.
>> No.73613 Reply
>>73598
А, я понял в чем моя проблема: я не понял, что такое ограниченная первообразная. А так я пробовал через Дирихле тоже.
>> No.73619 Reply
>>73600
> А в каком порядке?
В перечисленном. Прочитаешь эти книги - тогда и поговорим.
>> No.73628 Reply
>>73605
Допустим, ты хочешь заявить, что функция f(x) равна, по определению, скажем, |x+5|. Тогда ты пишешь, что f(x):=|x+5|. Затем заявляешь, что у тебя, скажем, по определению, g(x):=x+5. Тогда и f и g будут равны при x>-5, однако уже не потому, что ты это объявил (по определению), а по условию, то бишь если x>-5, то f(x)=g(x).
>> No.73633 Reply
>>73605
Различие тонкое. Знак a=b означает, что если у тебя есть некоторое выражение, содержащее a, то на место символа a ты можешь подставить символ b, и при этом выражение не изменится. А знак := означает, что ты вводишь новое понятие, смысл которого подлежит разъяснению.
>> No.73644 Reply
>>73600
нирикамендую читать кучу макулатуры, там внимания заслуживает только Зорич, Алексеев и Арнольд
а тем, кто нихуя не знает, надо читать "Что такое математика" Куранта Роббинса первой книжкой, очень она приятная
>> No.73652 Reply
>>73619
> Прочитаешь эти книги - тогда и поговорим.
Обязательно надо было выебнуться, да?
>> No.73654 Reply
>>73652
Да.
>> No.73656 Reply
>>73652
Я был груб, потому что ты не сказал спасибо и начал спрашивать про порядок чтения, что является глупым вопросом - все книги переплетены друг с другом, так как образуют фундамент математики. Лучше бы ты вместо разговоров уже начал читать. Время, оно ведь стоит дорого.
>> No.73659 Reply
>>73656
Конечно, глупый вопрос, ведь это для начинающих темный лес. Что тут такого?
Список книг дал, как будто одолжение сделал.
Спасибо.
>> No.73665 Reply
File: 0c2d6f893f311fcadc0b2c16702675ac.jpg
Jpg, 82.72 KB, 624×867 - Click the image to expand
edit Find source with google Find source with iqdb
0c2d6f893f311fcadc0b2c16702675ac.jpg
>>73659
Всегда пожалуйста, доброняша. Когда появятся вопросы - спрашивай, ответим.
>> No.73692 Reply
Аноны-чистые-математики, а чем вы на жизнь зарабатываете?
Просто работаете в близкой области: в лаборатории, программистом и т.п.?

И это, я подумал тут.
Есть действительные числа, геометрически расположенные на числовой оси; в одном измерении.
Есть комплексные, геометрически расположенные на числовой плоскости; в двух измерениях.
А есть ли такие числа, расположенные в трёхмерном пространстве или вообще в n-мерном?

Школьник-кун
>> No.73695 Reply
>>73692
Гиперкомплексные числа. 010100.65, чистый математик, работаю математиком, инженером, программистом, преподом.
>> No.73700 Reply
>>73692
i lol'd
А чем зарабатывают на жизнь чистые-врачи-кардиологи или чистые-водители-автобуса или чистые-ну-ты-понел?
Работают в близких областях, наверное.
>> No.73706 Reply
Изучаю «Теорему Абеля» Алексеева.
Есть вопрос по пикрилэйтед-заданию.
Неужели все композиции симметрий в таблице умножения кроме вида «cc», «dd» и «f*f» в итоге получаются равными «с»? Или я не дошёл пока до истины™?
>> No.73709 Reply
File: Симметрии.png
Png, 71.82 KB, 452×585 - Click the image to expand
edit Find source with google Find source with iqdb
Симметрии.png
Реквестирую прощение, не залил.
>> No.73714 Reply
>>73692
Ну а ответ на второй вопрос не так прост. Если коротко, то такие числа существуют, их великое множество, но они и не совсем числа. Называются эти конструкция алгебрами Клиффорда.
А если тебе хочется, чтобы, подобно комплексным числам, они обладали "нормальным" умножением (хотя бы ассоциативным и с делением), то есть теорема, что над действительными числами существует всего три тела (они же ассоциативные алгебры с делением): сами действительные числа, комплексные и кватернионы.
>> No.73717 Reply
>>73692
А, вообще, можно и без Клиффорда, можно сказать ещё проще: берёшь образующие константы (т.е. просто символы), определяешь на них для каждой пары умножение так, чтобы оно удовлетворяло требованиям, каким тебе хочется (коммутативность, ассоциативноть и т.д.). Т.е. каждой упорядоченной паре констант определяешь, чему равно их произведение, выраженное суммой опять же констант из набора с коэфициентами, какими сам хочешь. А потом на структурные константы натягиваешь действительное векторное пространство.
Но, как я выше написал, как бы ты ни старался, если хочешь ассоциативное и с делением, то кроме кватернионов, комплексных и действительных чисел ничего не получится.
>> No.73732 Reply
>>73692
Типа н-мерный вектор с вещественными координатами?
>> No.73746 Reply
>>73732
Типа теорема Фробениуса говорит, что н может быть сильно не любым.
>> No.73749 Reply
>>73692
> А есть ли такие числа, расположенные в трёхмерном пространстве
Тащемта нет, зато есть, например, в четырёхмерном, восьмимерном и шестнадцатимерном:
ruwiki://Кватернион
ruwiki://Числа_Кэли
ruwiki://Седенион
Процедура построения:
ruwiki://Процедура_Кэли_—_Диксона

Статья о сабже, где описаны поиски триплетов:
http://mathscinet.ru/books/algebra/index.php?help=0
>> No.73750 Reply
>>73749
А как называются числа 32 и 64-мерной алгебры?
>> No.73766 Reply
>>73749
наконец-то я узнал, как по-научному называется удвоение кольца с сопряжением!
> > Процедура Кэли — Диксона
====
>>73746
а ещё я узнал имени кого эта теорема, которую я цитировал выше
> > теорема Фробениуса
и от имиджборд бывает польза, lol
>> No.73771 Reply
>>73644
> там внимания заслуживает только Зорич, Алексеев и Арнольд
Что ещё сбуробишь?
>> No.73815 Reply
File: sum.png
Png, 0.80 KB, 101×50 - Click the image to expand
edit Find source with google Find source with iqdb
sum.png
Вычислить сумму.
>> No.73816 Reply
>>73815
Ты опять за своё, доброняш?
>> No.73822 Reply
>>73816
Ну, не с кем мне больше порешать.
>> No.73855 Reply
File: 1.png
Png, 1552.58 KB, 1253×1120 - Click the image to expand
edit Find source with google Find source with iqdb
1.png
>>73498
> он смотрится по меньшей мере странно среди тредов про ...
Почему же? Примерно одной возрастной категории проблемы ...
>>73815
Смотри-ка, анон, у меня, кажется, свернулось. План такой: арктангенс в степенной ряд, переставляем суммирования, схлопываем сумму по n в дзета-функцию Римана, выражаем ее значения в четных точках через числа Бернулли, и угадываем в полученном выражении степенной ряд для функции f(x) в точке ((корень из двух) умножить на пи), где f(0)=0, xf'(x)=(xctgx+ixctgix)/2. Ну есть известный ряд для xctgx через числа Бернулли, вот его и угадываем, фактически, только он в два раза разрежен. Кто проще?
>> No.73860 Reply
>>73855
На всякий случай добавлю, что это всё ещё фиг обоснуешь. С самого начала фиг знает, можно ли переставлять суммы; потом в конечном счёте получается ряд, мягко говоря, расходящийся, и мы интегрируем функцию с разрывом, в котором неопределенный интеграл не непрерывен, ну и прочие радости.
>> No.73865 Reply
>>73815
А разве она существует?
>> No.73868 Reply
>>73865
arctg(x)~x, а сумма единицанаквадратов сходится. Вроде норм ...
>> No.73878 Reply
File: 1251914005985.jpg
Jpg, 126.87 KB, 465×600 - Click the image to expand
edit Find source with google Find source with iqdb
1251914005985.jpg
>>60924
Удваиваю вопрос этого куна.
>> No.73939 Reply
О, кстати!
Тут же, наверное, есть студенты с мехмата и матмеха.
А вот скажите, в атмосфере ваших заведений витает что-то в духе:

"западная наука в кризисе, а у нас теплятся остатки разума"

"теория категорий/алгебраическая геометрия/гомологическая алгебра (нужное подчеркнуть) нужна только для того, чтобы получать гранты и мы её изучать не будем"

"современная математика вообще потеряла всякий смысл с реальностью и нам это не нужно"

ну и т.д.
>> No.73957 Reply
>>73939
Нет.
>> No.73959 Reply
>>73939
Хипстор с матфака детектед.
>> No.74071 Reply
File: Снимок-экрана-2012-08-17-в-13.04.03.png
Png, 27.17 KB, 752×266 - Click the image to expand
edit Find source with google Find source with iqdb
Снимок-экрана-2012-08-17-в-13.04.03.png
Анон, как максимально точно подобрать такие два ряда чисел, чтобы при делении каждого члена первого ряда на каждый член второго, получась бы арифметическая прогрессия? Пикрелейтед.
>> No.74073 Reply
>>74071
Взять в качестве первого ряда арифметическую прогрессию, а в качестве второго - ряд из одной константы, лол.
>> No.74077 Reply
>>74073 Это очевидно, да, но мне повторы чисел не нужны.
>> No.74079 Reply
File: tolsto.jpg
Jpg, 10.11 KB, 350×171 - Click the image to expand
edit Find source with google Find source with iqdb
tolsto.jpg
>>73939
Сначала я хотел что-нибудь написать, но потом вспомнил про картинку.
>> No.74080 Reply
>>74077
Взять в качестве первого ряда арифметическую прогрессию в квадрате, а в качестве второго - арифметическую прогрессию, лол.
>> No.74081 Reply
>>74071
Берёшь произвольный ряд - это второй. Умножаешь его члены на члены требуемой арифметической прогрессии - получаешь первый.
>> No.74082 Reply
>>74081
И, естесственно, в первом не должно быть нулей.
>> No.74083 Reply
>>74082
Тфу, во втором т.е. Совсем запутался.
>> No.74085 Reply
>>74079
ну почему же, я неоднократно встречал работающих в России людей, считающих себя математиками, для которых теория категорий (она почему-то их особенно пугает) представляется не иначе как инструментом для получения грантов. То есть, по их мнению западные математики специально договорились между собой, напридумывали всяких наукообразных теорий и требуют за это деньги, хотя сами они, западные математики, прекрасно понимают, что это липа и ведут себя тем самым нечестно. а вот мы гордо от этого отказываемся и будем заниматься тем, что нам завещали Лузин, Лупанов ну и т.д.
>> No.74093 Reply
File: Снимок-экрана-201...
Png, 33.41 KB, 793×273
edit Find source with google Find source with iqdb
Снимок-экрана-2012-08-17-в-15.40.46-(2).png
File: Снимок-экрана-201...
Png, 32.68 KB, 772×279
edit Find source with google Find source with iqdb
Снимок-экрана-2012-08-17-в-15.52.14.png

>>74080
>>74081 В 1 и 2 ряду по 4 числа, в третьем 16, что на что умножаем то? Домножил на 1; 1,5; 2; 2,5. Что-то похожее на правду.
Оба способа работают конечно, но как-то не оче.
И обоих случаях все очень сильно коверкается от выбранного произвольного шага прогрессии, тут или я дурак или все не так уж и просто.
>> No.74098 Reply
File: lolwut.jpg
Jpg, 70.71 KB, 533×594 - Click the image to expand
edit Find source with google Find source with iqdb
lolwut.jpg
>>74093
> В 1 и 2 ряду по 4 числа, в третьем 16, что на что умножаем то?
Что за ряды такие? Пикрелейтид.

Вообще, я так понял, вопрос не про ряды, а про последовательности

Прогрессия - an = k+n*m , де k,m из R, m != 0
Второй ряд - bn = f(n), где f(n) != 0 , для любого n
Первый ряд - cn = an * bn
Всё, для любого n выполняется cn / bn = an.
>> No.74103 Reply
File: index.jpg
Jpg, 35.21 KB, 525×340 - Click the image to expand
edit Find source with google Find source with iqdb
index.jpg
Итак, матанцы, возвращаюсь к вопросу об изучении матана самостоятельно. Путем размышлений, пришел к такой схеме последовательного усвоения материала:
1. Школьный курс математики и геометрии (желательно старые учебники издательства "Просвещение")
2. М.Я.Выгодский. Справочник по элементарной математике - для закрепления изученного.
3. Д.Письменный. Конспект лекций по высшей математике - плавный переход к Истинному Знанию.
4. М.Я.Выгодский. Справочник по высшей математике - вновь для закрепления.
5. Фихтенгольц, 3 тома - самая мякотка.
Дальше - хуй знает, что. Поправьте, подскажите. Но чтобы последовательность не терялась.
>> No.74105 Reply
>>74103
всем, кто хочет обсудить вопрос самостоятельного изучения, нужно первым делом писать, зачем им это надо, иначе содержательного срача обсуждения не получится.
====
Кстати, куда там пропал казах, который тоже собирался постигать науку? Выходи на связь.
>> No.74109 Reply
>>74103
Для начала напиши с какой целью ты изучаешь математику: подготовка к поступлению, необходимость в работе, хочешь знать чем занимается современная математика, хочешь начать заниматься математикой или что-то другое?
>> No.74110 Reply
>>74103
Зачем нужен "плавный переход"? Ведь "самая мякотка" вполне самодостаточна.
>> No.74116 Reply
>>74103
Во-первых, вряд ли без практики у тебя что-то получится. Во-вторых, для изучения матана, я боюсь, тебе понадобится изучить (или изучать параллельно) линейную алгебру. В-третьих, ничего у тебя не выйдет. Если ты сейчас даже школьную программу не знаешь, то матан точно сам не выучишь.
>> No.74118 Reply
>>74109
Изучение математики с азов до уровня, позволяющего вести научную деятельность.
>>74116
Меньше скепсиса, товарисч. Москва не сразу строилась.
Есть предложение по изменению схемы? Велкам.
>> No.74124 Reply
>>74118
> Есть предложение по изменению схемы?
http://imperium.lenin.ru/~verbit/MATH/programma.html
Это без аспирантуры и постдока. И лет через восемь будешь вести научную деятельность.
>> No.74128 Reply
>>73878
Поднимаю свой вопрос снова.
>> No.74159 Reply
>>74118
> Изучение математики с азов до уровня, позволяющего вести научную деятельность.
Тогда не читай ничего из тобой предложеного. Можешь почитать книжки отсюда >>74124 хотя если уровень совсем низкий, тогде-то попробуй >>73546. И будет трудно достичь нормального уровня без учебы в вузе на мат-специальности. Ну и вообще читай лучше сразу вузовскую программу, причем математическую а не инженерную.
>> No.74208 Reply
>>74128
Хм. Желающих изучать математику становится всё больше. Наверное, пора серьёзно взяться за эту проблему и разработать программу курса современной математики. Будем думать, будем серьёзно думать.
>> No.74215 Reply
>>74208
Надо бы рассказать про подготовку к ЕГЭ.
И тред пересоздать.
>> No.74217 Reply
>>74215
> пересоздать
Сделано.

Следующий: >>74216


Password:

[ /tv/ /rf/ /vg/ /a/ /b/ /u/ /bo/ /fur/ /to/ /dt/ /cp/ /oe/ /bg/ /ve/ /r/ /mad/ /d/ /mu/ /cr/ /di/ /sw/ /hr/ /wh/ /lor/ /s/ /hau/ /slow/ /gf/ /vn/ /w/ /ma/ /azu/ /wn/ ] [ Main | Settings | Bookmarks | Music Player ]